Categories
Exam Questions Harvard

Harvard. Exams for Principles of Accounting. W.M. Cole, 1905-06

The principles of accounting course taught by William Morse Cole in the department of economics at Harvard was expanded to two semesters in 1905-06. 

__________________________

Earlier accounting exams

About William Morse Cole
1901-02
1902-03
1903-04
1904-05

__________________________

Course Enrollment
1905-06

Economics 18. Mr. W. M. Cole. — Principles of Accounting.

Total 44: 6 Graduates, 23 Seniors, 7 Juniors, 3 Sophomores, 5 Others.

Source: Harvard University. Report of the President of Harvard College, 1905-1906, p. 73.

__________________________

ECONOMICS 18
Mid-year Examination, 1905-06

I
  1. What is the function of nominal accounts in double-entry book-keeping?
  2. The books which the following trial balance represents are correct and record only normal transactions. Is the trial balance satisfactory? If not, show how you would go to work to correct it.
Proprietor 45,000
Merchandise 17,200
Cash 9,000
Interest 500
Bills Payable 4,000
Real Estate 15,000
Bills Receivable 20,000
Accounts Receivable 14,000
Rentals 500
Expenses 8,200
______ ______
66,700 66,700
  1. If rent is earned on buildings owned, should the amount of rent be debited or credited? Is it objectionable to enter the amount to the Real Estate account? If so, why?
  2. Construct a special-column journal to do all the work of journal, sales book, purchase book, and special-columncash book, combined. Illustrate briefly how posting would be done from it.
  3. Name the four common groups of railroad operating-expense accounts and show the logic of the classification.
II

            Show the balance-sheet for a business which meets the following conditions: Capital stock, 200,000; cash on hand, 7,000; surplus, 50,000; manufactured goods on hand, 10,000; notes outstanding, 25,000; sums owed for raw material, 25,000; sums owed for wags, 3,000; raw material on hand, 6,000; undivided profits, 4,000; notes of customers on hand, 17,000; depreciation fund, in bonds, 8,000; sums due from customers, 15,000; real estate, 100,000; machinery, etc., 144,000.

            In the following year the business is as follows: Goods manufactured (on contract), 147,000 (contract price); goods delivered (on contract), 135,000; collected on goods, 129,000 in cash, 20,000 in notes; labor expense incurred and paid, 50,000; raw material bought, 75,000; raw material paid for, 90,000; raw material consumed, 70,000; new machinery bought and paid for, 20,000; interest paid, 1,000; interest accrued against the corporation, not yet due, 200; collected on notes, 28,000; general expenses paid, 9,000; borrowed on notes, 50,000; repairs of buildings paid from depreciation fund, 2,000; paid on notes, 20,000; losses from bad debts, 1,000; taxes, 1,000; dividends, 17,300.

            Show the income sheet for the year.

            Show the balance sheet for the new year.

            [It is probable that time will be saved and confusion will be avoided if a rough journal and a rough ledger are used for assistance in working out the above problem. Only results are required, however.]

Source: Harvard University Archives. Harvard University. Mid-year Examinations, 1852-1943. Box 7, Bound Volume: Examination Papers, Mid-Years 1905-06.

__________________________

ECONOMICS 18
Year-end Examination, 1905-06

Follow the order of the paper, not only in arrangement but also in performance.

  1. What is a trial balance? How far is it useful?
  2. What is the relation between an income sheet and a balance sheet? Can an item appear upon both at the same time?
  3. What ought books to show with regard to discounts (on merchandise) for prompt payments?
  4. Arrange the following items of a railroad report in proper form, and fill in any important omissions by figures derived from the items given:—
[Figures represent millions of dollars]
Fixed charges 5.0 Accrued liabilities 3.0
Accounts payable 4.0 Claims audited 1.0
Bills receivable 0.5 Funded debt 85.0
Other income 4.0 Supplies 3.0
Investments 67.0 Accounts receivable 4.0
Accrued assets 1.5 Earnings from operation 17.7
Dividends 1.8 Cost of road 50.0
Cash 4.0 Net earnings 5.9
Capital stock 30.0 Miscellaneous assets 1.0
Betterments charged to income 1.0
  1. A partnership is organized on January 1. On July 1 a partner dies. How far is a correct trial balance taken on the latter day a satisfactory basis for a settlement of the dissolved partnership? Assuming the ordinary commercial accounts to be kept, how should you go to work to determine the share of each partner?
  2. Except for the items mentioned below, a corporation’s balance sheets for 1905 and 1906 show the same figures. How much do these items tell about the history of the corporation for the year 1905-1906?

 

1905
Surplus $70,000

1906

Depreciation Fund $20,000 Reserve Fund $60,000
Depreciation Fund $20,000
Surplus $10,000
  1. How should you determine on an interest date the value of ten-year bonds bought (at the time of issue) at a premium and now having five years more to run? [Do not figure, but describe have process.] Should the books show that value? If so, how and where? If a portion of the bonds is now sold at less than the value so determined, how should the sale show on the books?
  2. A factory is engaged in a dying industry. Five years is the estimated life of the machinery and of the good-will. The buildings are convertible to other uses. The corporation will be dissolved at the end of five years.
    Describe a brief administrative policy that you would recommend to close the business and end the corporation, and then describe the accounting processes to record the closing transactions and to leave no balances on the books.

Source: Harvard University Archives. Harvard University, Examination Papers 1873-1915. Box 8, Bound volume: Examination Papers, 1906-07; Papers Set for Final Examinations in History, Government, Economics,…,Music in Harvard College (June, 1906), pp. 42-43.

Image SourceHarvard Alumni Bulletin, Vol. XIX, No. 16 (January 18, 1917), p. 308. Portrait of William Morse Cole colorized by Economics in the Rear-view Mirror.

Categories
Exam Questions Harvard

Harvard. Accounting principles. Enrollment and final exam. Cole, 1904-1905

“Principles of Accounting” was one of three courses offered by the department of economics that were specifically targeted to advanced students who intended to start business careers after graduation. William Morse Cole, A.M. was the instructor. An earlier post provides an overview of his career.

__________________________

From previous years…

1900-01
1901-02
1902-03
1903-04

__________________________

Course Enrollment
1904-05

Economics 18 1hf. Mr. W. M. Cole. — Principles of Accounting.

Total 27: 7 Graduates, 14 Seniors, 2 Juniors, 1 Sophomore, 3 Others.

Source: Harvard University. Report of the President of Harvard College, 1904-1905, p. 75.

__________________________

Course Description
1904-05

[Economics] 18 1hf. The Principles of Accounting. Half-course (first half-year). Mon., Wed., and (at the pleasure of the instructor) Fri., at 3.30. Mr. W. M. Cole.

This course is designed primarily for students who expect to enter a business career, and wish to understand the processes by which the earnings and values of industrial properties are computed. It is not intended to afford practice in book-keeping, but to give students a grasp of principles which shall enable them to comprehend the significance of accounts.

In order that students may become familiar with book-keeping terms and methods, a few exercises will be devoted to a brief study of the common systems of recording simple mercantile transactions. The chief work of the course, however, will be a study of the methods of determining profit, loss, and valuation. This will include an analysis of receipts, disbursements, assets, and liabilities, in various kinds of industry, and consideration of cost of manufacture, cost of service, depreciation and appreciation of stock and of equipment, interest, sinking funds, dividends, and the like. Published accounts of corporations will be studied, and practice in interpretation will be afforded. Attention will also be given to the functions and methods of auditors.

The instruction will be given by lectures, discussions, reading, and written work.

Course 18 is open to Seniors and Graduates who have taken Economics 1.

Source: Harvard University. Faculty of Arts and Sciences. Division of History and Political Science Comprising the Departments of History and Government and Economics, 1904-05 (May 16, 1904), pp. 47-48.

__________________________

ECONOMICS 18
Mid-year Examination, 1904-05

  1. Construct an imaginary trial balance of at least eight items, of which not over three shall represent persons or corporations.
  2. In a proprietor’s absence the books of a business are opened and kept by a bookkeeper who keeps accurate record of transactions reported to him but cannot be trusted to figure valuations or profits. At the end of a year, the records show, before the books are closed and simply as the result of regular transactions, the following figures:
    Proprietor’s investment, $100,000; Bills Payable, $17,000; Bills Receivable, $26,000; Real Estate, $20,000; Accounts Payable, $15,000; Accounts Receivable, $20,000; Cash, $5,000; Merchandise on hand, valued at cost, $75,000; Merchandise Dr. on ledger, $49,500; Expense, $12,000; Interest balance received, $500.
    Now the proprietor returns and wishes to close his books for the year. If he needs any information not given above, what questions will he ask in obtaining it? Assume any fairly reasonable answers to such questions, if any, and then show what is the proprietor’s present investment in the business.
  3. The annual report of a corporation shows the following figures: Profit and Loss credit balance at the beginning of the year, $20,000; Merchandise profit, $140,000; Expense, $100,000; appreciation of Real Estate, $10,000; permanent Surplus set aside at the end of the year, $15,000; Dividends, $50,000. Present in rough ledger form i.,e. debit and credit items, the Profit and Loss account for the year, showing the balance at the end of the year.
  4. Explain the significance of each of the following accounts appearing, with a balance on the side indicated, on (1) a balance sheet for the beginning of a new fiscal year, and (2) a trial balance taken in the ordinary course of business during the year: Wages, Cr.; Interest, Dr.; Merchandise, Dr.; Profit and Loss, Dr.; Reserve Fund, Cr.; Supplies, Dr.
  5. An annual report of a corporation shows, either on the income sheet or on the balance sheet, an item of which the title means little or nothing to you, e.g., “Pro rata share of bonds of H.R.L. Co.” Assuming the accounts to be kept properly, explain to what extent the position and the treatment of the item throw light upon its character.
  6. A corporation shows the following statements:—
1903. 1904.
Plant $450,000 $440,000
Supplies 59,000 27,000
Real Estate 100,000 95,000
Cash 15,000 15,000
Bills Receivable 50,000 60,000
Accounts Receivable 20,000 15,000
Merchandise 83,000 108,000
Capital Stock 500,000 500,000
Bills Payable 150,000 150,000
Accounts Payable 100,000 90,000
Wages 7,000 10,000
Profit and Loss 20,000 10,000
Proceeds from sales 600,000
Direct cost of production 500,000
General expenses, fixed charges, etc. 70,000

Tell all you can about the history of the corporation for the year 1903-04.

  1. If bonds on hand were bought at a premium, should that fact show upon the income sheet or in any way affect it?
  2. The books of three concerns are open to your inspection. Outline briefly a scheme for consolidating the three concerns into a corporation.
    [Do not allow yourself to get involved in the fascinating details of such a scheme at the expense of time needed for other things. This may well be left for surplus time and energy. Bookkeeping entries are not required. Show how the accounts of the three concerns can be interpreted for such a purpose.]

Source: Harvard University Archives. Harvard University, Examination Papers 1873-1915. Box 7, Bound volume: Examination Papers, 1904-05; Papers Set for Final Examinations in History, Government, Economics,…,Music in Harvard College (June, 1905), pp. 37-38.

Image SourceHarvard Alumni Bulletin, Vol. XIX, No. 16, p. 308. Portrait of William Morse Cole colorized by Economics in the Rear-view Mirror.

Categories
Exam Questions Harvard

Harvard. Final exam for Principles of Accounting. Cole, 1903-1904

William Morse Cole offered his principles of accounting course as a vocational non-credit course for seniors in the Harvard economics department from 1900-1905, after which time the subject received for-credit status. In 1908 Cole was appointed assistant professor of accounting in Harvard’s newly established business school.

Exam questions for 1900-01, 1901-02 and 1902-03 have been posted earlier.

___________________________

ECONOMICS 18
Course enrollment. 1903-04

Economics 18 1hf. Mr. W.M. Cole. — The Principles of Accounting.

Total 51: 2 Graduates, 33 Seniors, 12 Juniors, 1 Sophomore, 3 Others.

Source: Harvard University. Report of the President of Harvard College, 1903-1904, p. 67.

___________________________

ECONOMICS 18
Mid-Year Examination. 1903-04

I.

Do not devote more than half your time to this part of the paper.

  1. (a) On which side of a balance sheet is the balance of the following accounts likely to be?
    (b) What, in each case, does the amount of such a balance indicate?

Bills Receivable.
Accounts Payable.
Plant.
Profit and Loss.
Wages.
Merchandise.

  1. If December 31 is the end of the fiscal year, and after office hours on that day the books are closed for the year, in what respects would a trial balance, taken at the opening of business on January 1, differ from one taken at the close of business December 31? Illustrate by means of specific accounts.
  2. Why, in any logical system of accounting, must there be a debit for every credit?
  3. (a) What is the function of a ledger? (b) How do items usually get into a ledger? (c) Can items in the ledger have any other origin than that referred to in (b)? If so, what?

II.

  1. Suppose you are the executor of the estate of one of two partners. Suppose you are given a trial balance of the books as they stood at the time of the death of the partner whose estate you represent, and that the surviving partner swears to the correctness of that trial balance. In order to be sure of doing justice to the heirs of the deceased partner, how far do you need, in arranging settlement, to go beyond or behind the trial balance?
  2. You organize a corporation, and on January 1, 1904, the following facts are shown by the books:—

The corporation has taken over from an individual owner a business of which the assets, determined by conservative valuation of the property, are $100,000 (Bills Receivable and Accounts Receivable, $20,000; Supplies, $5,000; Real Estate and Plant, $75,000). Capital stock to the amount of $500,000 has been issued, of which $200,000 has been given the original owner for his title, $300,000 has been sold for cash at par. The corporation has bought a neighboring plant for $100,000, paying for it by Bill Payable to that amount.

Show a brief balance sheet under these conditions.

Now, you are absent from the corporation’s affairs for two years. On your return you are told that 6 per cent. dividend has been paid in each year, and you are shown the balance sheets below.

Write a brief history of the business for each of the two years of your absence.

Jan. 1, 1905.

Real Estate and Plant, $420,000 Capital Stock $500,000
Bills Rec.& Accts.Rec. 70,000 Funded Debt 100,000
Supplies 5,000 Profit and Loss 20,000
Merchandise 105,000
Cash 20,000            
$620,000 $620,000

Jan. 1, 1906.

Real Estate and Plant $400,000 Capital Stock $600,000
Deprec’n Fund Bonds 20,000 Reserve Fund 20,000
Reserve Fund Bonds 20,000 Profit and Loss 20,000
Bills Rec.& Accts.Rec. 70,000
Supplies 5,000  
Merchandise 105,000  
Cash 20,000            
$640,000 $640,000
  1. (a) What is the difference between a revenue and a capital account?
    (b) If time shows that items originally charged to revenue account should have been charged to capital account, what treatment can be given such items at the end of the year, without changing their ledger classification, so that the profit or the loss shall be correctly determined?
    (c) Reverse the conditions of (b), — i.e., assume error to have been made in charging to capital instead of to revenue, — and designate the required treatment.
  2. Conceive the balance sheet of a railroad to be exactly the same for 1904 as for 1903. Conceive the true net income to be $11,000,000, the fixed charges $3,000,000, the “other income” $2,000,000, and the road to be operated for 66 2/3 per cent.
    Construct as much as you can of the income sheet.

Source:  Harvard University Archives. Harvard University, Mid-year examinations 1852-1943. Box 7, Bound volume: Examination Papers, Mid-Years, 1903-04.

 

 

Categories
Exam Questions Harvard

Harvard. Principles of Accounting. Course description, enrollment, and final exam. W.M. Cole, 1902-1903

As the course description clearly indicates, this undergraduate accounting course was offered by the economics department for those Harvard students planning a business career. At the time accounting was seen to be a kissing cousin to statistics and both essentially amounted to a hill of bean-counting.

______________________

Description of Economics 18
First term, 1902-03

  1. 1hf. *The Principles of Accounting. Half-course (first half-year). Mon., Wed., and (at the pleasure of the instructor) Fri., at 3.30. Mr. W. M. Cole.

This course is designed primarily for students who expect to enter a business career, and wish to understand the processes by which the earnings and values of industrial properties are computed. It is not intended to afford practice in book-keeping, but to give students a grasp of principles which shall enable them to comprehend the significance of accounts.

In order that students may become familiar with book-keeping terms and methods, a few exercises will be devoted to a brief study of the common systems of recording simple mercantile transactions. The chief work of the course, however, will be a study of the methods of determining profit, loss, and valuation. This will include an analysis of receipts, disbursements, assets, and liabilities, in various kinds of industry, and a consideration of cost of manufacture, cost of service, depreciation and appreciation of stock and of equipment, interest, sinking funds, dividends, and the like. Published accounts of corporations will be studied, and practice in interpretation will be afforded. Attention will also be given to the functions and methods of auditors.

The instruction will be given by lectures, discussions, reading, and written work.

Course 18 is open to Seniors and Graduates who have taken Economics 1.

Source: Harvard University. Faculty of Arts and Sciences, Division of History and Political Science  [Comprising the Departments of History and Government and Economics], 1902-03. Published in The University Publications, New Series, no. 55. June 14, 1902.

______________________

Enrollment in Economics 18
First term, 1902-03

Economics 18. 1hf. Mr. W. M. Cole. — The Principles of Accounting.

Total 46: 1 Gr., 28 Se., 11 Ju., 3 So., 3 Others.

Source: Harvard University. Annual Report of the President of Harvard College, 1902-03, p. 68.

______________________

Final Exam in Economics 18
First term, 1902-03

ECONOMICS 18
  1. Under normal conditions, on which side of a ledger — debit or credit — will appear the balance of the following accounts? In each case, state why you think as you do.

Bills Receivable.
Bills Payable.
Capital Stock.
Expense.
Accounts Payable.

  1. What should be debited and what credited after each of the following transactions?
    1. Buying on credit, at the first of the year, stationery expected to last eight months.
    2. Paying for that stationery by issuing a note.
    3. Paying that note.
    4. Exchanging that stationery at cost (none being used) for merchandise.
    5. Selling that merchandise at cost and receiving a note in payment.
    6. Collecting cash for the note.
      Now what is the net result, upon ledger balances, of all these transactions?
  2. Of the following transactions what would be the ultimate effect upon a railroad balance sheet? Designate, in each case, on which side of the balance sheet, and in what items, the change would appear.
    1. The issue of new capital stock and the use of the proceeds for new (additional) equipment.
    2. A conversion of bonds into stock.
    3. A distribution of accumulated profits of the past in the form of a scrip dividend which is converted into stock.
    4. A reduction in the valuation (set by the company in its own books) of stocks and bonds owned.
    5. Watering stock to represent supposed increase in earning capacity.
  3. In the middle of a business year, July 1, the sole proprietor of a store dies suddenly. You, as his executor, must determine the exact worth of his business. The trial balance of July 1 is given you. Can you get all needed information from that trial balance? If not, what is lacking? State just what you would do to determine the worth of the business. If you can explain best by figures, assume arbitrary figures (not necessarily reasonable) and proceed with those as a basis. Processes, rather than results, are to be shown.
  4. In cases of depreciation, one of at least three courses is open to the managers. State what are the three, and show how each is treated in the accounts.
  5. The following is a page of a book:–
Jan. 1 Balance 1,547.30
A. Andrews His invoice, Dec. 1 615.10
Bills receivable No. 127 paid 500.00
Bills payable No. 19 discounted 1,000.00
Merchandise Cash sales 173.28
Jan. 2 Bills receivable No. 116 paid 123.50
Insurance Premium ret’d on policy 73.23
Jan. 3 Bills receivable No. 139 paid 312.26
Bills receivable 935.76 935.76
Cash, Dr. 2,797.37 2,797.37
4,344.67

Explain fully what transactions are here recorded. What postings should be made, and to which side of each account? If any figures are not to be posted, why not?

  1. Can all the accounts of a business be nominal? Why, or why not?
    Can all the accounts of a business be real? Why, or why not?

Source: Harvard University Archives. Mid-year Examinations 1852-1943. Box 6. Papers (in the bound volume Examination Papers Mid-years 1902-1903).
Also included in Harvard University Archives. Examination Papers 1873-1915. Box 6. Papers Set for Final Examinations in History, Government, Economics, History of Religions, Philosophy, Education, Fine Arts, Architecture, Landscape Architecture, Music in Harvard College, June 1903 (in the bound volume Examination Papers 1902-1903).

Image SourceHarvard Alumni Bulletin, Vol. XIX, No. 16, p. 308. Portrait of William Morse Cole colorized by Economics in the Rear-view Mirror.

Categories
Exam Questions Harvard

Harvard. Principles of Accounting. Exam questions. W. M. Cole, 1901-1902

Harvard. Life of accounting professor William Morse Cole, A.M. 1896

____________________________

Course Description
ECONOMICS 181
1901-1902

For upperclass undergraduates and graduates to prepare for a business career

The Principles of Accounting. Half-course (first half-year). Mon., Wed., and (at the pleasure of the instructor) Fri., at 3.30. Mr. W. M. Cole.

This course is designed primarily for students who expect to enter a business career, and wish to understand the processes by which the earnings and values of industrial properties are computed. It is not intended to afford practice in book-keeping, but to give students a grasp of principles which shall enable them to comprehend the significance of accounts.

In order that students may become familiar with book-keeping terms and methods, a few exercises will be devoted to a brief study of the common systems of recording simple mercantile transactions. The chief work of the course, however, will be a study of the methods of determining profit, loss, and valuation. This will include an analysis of receipts, disbursements, assets, and liabilities, in various kinds of industry, and a consideration of cost of manufacture; cost of service, depreciation and appreciation of stock and of equipment, interest, sinking funds, dividends, and the like. Published accounts of corporations will be studied, and practice in interpretation will be afforded. Attention will also be given to the functions and methods of auditors.

The instruction will be given by lectures, discussions, reading, and written work.

Course 18 is open to Seniors and Graduates who have taken Economics 1.

Source: Harvard University Archives. Official Register of Harvard University 1901-1902. Box 1. Faculty of Arts and Sciences, Division of History and Political Science (June 21, 1901), University Publications, New Series, No. 16, pp. 46-47.

____________________________

Course Enrollment
ECONOMICS 181
1901-02

Economics 181hf. Mr. W. M. Cole. — The Principles of Accounting.

Total 36: 1 Graduate, 24 Seniors, 5 Juniors, 2 Sophomores, 4 Others.

Source: Harvard University. Report of the President of Harvard College, 1901-1902, p. 78.

____________________________

Semester Final Examination
ECONOMICS 18
First Half-year 1901-1902

  1. What is the profit and loss account?
    Are charges or credits made direct to this account? If so, under what circumstances?
    How is this account closed?
  2. Formulate journal entries to express each of the following transactions :—
    1. a sale of goods for a note bearing interest;
    2. discounting the above note at a bank;
    3. annulling a personal account as uncollectible;
    4. interest charged, but not paid, on a long-standing account.
  3. What is indicated by each of the following ledger accounts when
    1. the account shows a debit balance, and
    2. the account shows a credit balance: loss and gain, merchandise, rent, commission, purchase ledger, stores?
  4. Illustrate roughly a columnar cash book and a columnar journal. Comment on the columnar system.
  5. Which of the following should be charged to capital account and which to revenue account: The purchase of a patent right; legal fees for organizing a corporation; the purchase of a lease; repairs of machinery; replacement of machinery; the purchase of additional machinery; the loss by fire of uninsured property?
    Explain in each case why you decide as you do.
  6. You contemplate purchasing an interest in a business that has run five years, and agree to pay one third the valuation of its net assets. The following statement is given you by the partners:—

Dr.

Buildings, machinery, etc., at cost $50,000
Expended for repairs and renewals $8,000
Patent rights purchased $14,000
Balance of sales ledger $26,000
Inventory, as per stores and stock books $19,000
$117,000

Cr.

Bills payable $48,000
Balance of purchase ledger $47,000
Partners’ capital $22,000
$117,000
    1. Assuming that this statement gives all that you need to know, how much must you pay for your interest in the business?
    2. Does this statement give all necessary information about the assets? If not, what is lacking?

7.  The chief items on the balance sheet of a firm may be summarized as follows :—

Assets.
Cash, stock, and accts. receivable $48,000
Buildings and machinery $37,000
$85,000
Liabilities.
Notes and accts. payable $33,000
Capital of the partners $62,000
$95,000

Is the firm solvent or insolvent? Why do you think so?

  1. The books of a firm have been kept by the ordinary methods and all transactions prior to the close of business on December 31 have been carried through the books. You are asked to determine profit or loss for the year just closed. Describe carefully the steps that you would take.

Source: Harvard University Archives. Harvard University Mid-Year Examination Papers, 1852-1943. Box 6, Bound volume, Mid-year Examination Papers, 1901-02. Sub-volume Papers Set for Final Examinations in History, Government, Economics, … in Harvard College (January 1902). Also included in Harvard University Examination Papers, 1873-1915. Box 6, Bound volume, Examination Papers, 1902-03. Sub-volume Papers Set for Final Examinations in History, Government, Economics, … in Harvard College (June 1902).

Image SourceHarvard Alumni Bulletin, Vol. XIX, No. 16, p. 308. Portrait of William Morse Cole colorized by Economics in the Rear-view Mirror.

Categories
Exam Questions Harvard

Harvard. Economics semester final examinations, 1900-01.

In the first full academic year of the twentieth century the Harvard economics department offered the following courses. The course links take you to the official course announcement, instructor names, enrollment figures, and the transcribed semester examinations.

Economics 1. Outlines of Economics
Economics 2. Economic Theory of the 19th Century
Economics 3. Principles of Sociology
Economics 5. Railways and Other Public Works
Economics 6. Economic History of the U.S.
Economics 8. Money
Economics 9. Labor Question in Europe and the U.S.
Economics 10. European Mediaeval Economic History
Economics 12. Banking and Leading Banking Systems
Economics 12a. International Payments and Gold/Silver Flows
Economics 13. Methods of Economic Investigation
Economics 17. Economic Organization and Resources in Europe
Economics 18. Principles of Accounting
Economics 19. General View of Insurance
Economics 20d. Adam Smith and Ricardo

__________________________________

Economics 1.
Outlines of Economics

Primarily for Undergraduates.

Course Announcement
  1. Outlines of Economics. , Wed., Fri., at 9. Professor Taussig, Dr. Sprague, Mr. Andrew, and Messrs. — and — .

Source: Harvard University. Announcement of the Courses of Instruction provided by the Faculty of Arts and Sciences for the Academic Year 1900-1901, p. 41.

Course Enrollment
  1. Professor [Frank W.] Taussig, Drs. [Oliver Mitchell Wentworth] Sprague and [Abram Piatt] Andrew, and Messrs. [Charles] Beardsley and [James Horace] Patten. — Outlines of Economics.

Total 442: 23 Seniors, 70 Juniors, 257 Sophomores, 29 Freshmen, 63 Others.

Source: Harvard University. Report of the President of Harvard College 1900-1901, p. 64.

1900-01
ECONOMICS 1
[Mid-year Examination]

Arrange your answers strictly in the order of the questions.

  1. In what manner do you think that (a) the individual efficiency of laborers, (b) their collective efficiency, would be affected by the general adoption of profit sharing? of socialism?
  2. It has been said that the original formation of capital is due to abstinence or saving, but its permanent maintenance is not. What do you say to either statement?
  3. Wherein is Walker’s presentation of the forces that make the general rate of wages better than Mill’s, wherein not so good?
  4. “The extra gains which any producer or dealer obtains through superior talents for business, or superior business arrangements, are very much of a similar kind. …All advantages, in fact, which one competitor has over another, whether natural or acquired, whether personal or the result of social arrangements, bring the commodity, so far, into the Third Class, and assimilate the advantage to a receiver of rent.”
    Explain (a) what is this Third Class, and what is the law of value applicable to it; (b) what Mill would say as to the proposition here stated; (c) what Walker would say?
  5. What qualifications of the general principle of rent can you state, in its application to (a) premises used for building purposes, (b) dwelling-houses, (c) mines?
  6. If all men had the same start in life, would there be differences of wages? If so, of what sort? If not, why not?
  7. “Since cost of production here fails us, we must revert to a law of value anterior to cost of production and more fundamental…” In what cases does cost of production fail us? Will “cost of reproduction” cover such cases? Is there another law more fundamental?
  8. Under what circumstances. if ever, will a general rise in wages affect the relative values of commodities? Would your answer be the same as to a general rise in profits?
  9. In what manner do you believe business profits, interest, and wages would be affected by the general adoption of the various forms of consumers’ coöperation? of producers’ cooperation?

Source: Harvard University Archives. Harvard University Mid-year Examinations, 1852-1943. Box 4, Bound volume: Examination Papers, Mid-Years, 1900-01.

1900-01
ECONOMICS 1
[Year-end Examination]

I.
Answer three.

  1. How will the value of land be affected by
    1. an increase in population,
    2. a reduction in the rate of interest,
    3. a protective tariff on agricultural produce.
  2. How will the price of grain be affected by
    1. a tax proportioned to the economic rent of the land,
    2. an equal tax upon all land.
  3. “Profits do not form a part of the price of the products of industry, and do not cause any diminution of the wages of labor.”
    Would Mill agree to this statement? Would you?
  4. Upon what does the general level of wages depend (a) according to Mill, (b) according to Walker? What would you expect these writers to say as to the effect of a protective tariff on the general level of wages?

II.
Answer two.

  1. If a country exports on a large scale a commodity not previously exported, will its other exports be affected? If so, how? If not, why not?
  2. Can a country have a permanently “unfavorable” balance of trade? If so, under what conditions? If not, why not?
    Can a country permanently export specie? If so, under what conditions? If not, why not?
    Can the rate of foreign exchange in a country be permanently at the specie-shipping point? If so, under what conditions? If not, why not?
  3. How would you expect the issue of a paper currency to effect foreign trade,—
    1. While the notes were still redeemable;
    2. After they had become irredeemable.

III.
Answer two.

  1. Define the following terms

Seignorage,
Clearing house loan certificates,
Silver Certificates,
United States notes,
Inconvertible paper.

  1. How would the adoption of bimetallism affect the stability of the value of money?
    1. according to Mill,
    2. according to Walker,
    3. in your own opinion.
  2. How is the value of money in a country likely to be affected by an increase in
    1. the quantity of commodities produced and sold,
    2. the quantity of bank notes,
    3. the volume of bank deposits.

Which of these changes would you expect to exercise most influence? Which least? Give your reasons.

IV.
Answer all.

  1. Compare and explain the operations of the Bank of England and those of the New York banks in a time of crisis,
  2. Arrange these items…

Government Securities 40.
Surplus 3.
Notes 38.
Specie 40.
Deposits 55.
Capital 14.
Loans 30.

    1. … in their proper order, as they would stand in an account of the Bank of France.
    2. … as they would stand in an account of a national bank of the United States; and state (1) whether this could be an account of a national bank, and (2) whether the proportions of the different items are such as you would be likely to find in an account of such a bank.
    3. … as they would stand in an account of the Bank of England, assuming the uncovered issue to be 17.

Source: Harvard University Archives. Harvard University, Examination Papers, 1873-1915. Box 5, Bound volume: Examination Papers, 1900-01. Papers Set for Final Examinations in History, Government, Economics, Philosophy, Education, Fine Arts, Architecture, Landscape Design, Music in Harvard College (June, 1901), pp. 21-23.

 

__________________________________

Economics 2.
Economic Theory
in the 19th Century

For Undergraduates and Graduates.

Course outline and readings.

Course Announcement
  1. Economic Theory in the Nineteenth Century. , Wed., Fri., at 2.30 Professor Taussig. [note: Professor Carver taught the course]

Source: Harvard University. Announcement of the Courses of Instruction provided by the Faculty of Arts and Sciences for the Academic Year 1900-1901, p. 41.

Enrollment
  1. Professor Carver. — Economic Theory in the Nineteenth Century.

Total 45: 6 Graduates, 15 Seniors, 16 Juniors, 5 Sophomores, 3 Other.

Source: Harvard University. Annual Reports of the President and Treasurer of Harvard College, 1900-01, p. 64.

1900-01
ECONOMICS 2
[Mid-year examination]
  1. Define value and explain why one commodity possesses more value in proportion to its bulk than another.
  2. Explain the various uses of the term diminishing returns, and define it as you think it ought to be defined.
  3. In what sense does a law of diminishing returns apply to all the factors of production.
  4. State briefly Böhm-Bawerk’s explanation of the source of interest.
  5. What, if any, is the relation of abstinence to interest.
  6. Would you make any distinction between the source of wages and the factors which determine rates of wages? If so, what? If not, why not?
  7. Discuss the question: Is a demand for commodities a demand for labor?
  8. What is the relation of the standard of living to wages.
  9. Discuss briefly the following questions relating to speculators’ profits. (a) Do speculators as a classmake any profits? (b) Are speculators’ profits in any sense earned?
  10. In what sense, if any, does the value of money come under the law of marginal utility?

Source: Harvard University Archives. Harvard University Mid-year Examinations, 1852-1943. Box 4, Bound volume: Examination Papers, Mid-Years, 1900-01.

1900-01
ECONOMICS 2
[Year-end Examination]

Discuss the following topics.

  1. The bearing of the marginal utility theory of value upon the questions of wages and interest.
  2. The definitions of capital as given by Taussig and Clark.
  3. Clark’s explanation of the place of distribution within the natural divisions of economics.
  4. Clark’s method of distinguishing between the product of labor and the product of capital.
  5. Clark’s distinction between rent and interest.
  6. Böhm-Bawerk’s theory of the nature of capital.
  7. The origin of capital, according to Böhm-Bawerk and Clark.
  8. The meaning of the word “productive” in the following proposition: “Protection is an attempt to attract labor and capital from the naturally more productive, to the naturally less productive industries.”
  9. The incidence of tariff duties.
  10. The theory of production and the theory of valuation as the two principal departments of economics.

Source: Harvard University Archives. Harvard University, Examination Papers, 1873-1915. Box 5, Bound volume: Examination Papers, 1900-01. Papers Set for Final Examinations in History, Government, Economics, Philosophy, Education, Fine Arts, Architecture, Landscape Design, Music in Harvard College (June, 1901), pp. 23-24.

 

__________________________________

Economics 3.
Principles of Sociology

For Undergraduates and Graduates.

Course Announcement
  1. The Principles of Sociology. — Development of the Modern State, and of its Social Functions. , Wed., and (at the pleasure of the instructor) Fri., at 1.30. Mr. —.

Source: Harvard University. Announcement of the Courses of Instruction provided by the Faculty of Arts and Sciences for the Academic Year 1900-1901, p. 41.

Enrollment
  1. Asst. Professor [Thomas Nixon] Carver. — The Principles of Sociology. Development of the Modern state, and of its Social Functions.

Total 57: 9 Graduates, 22 Seniors, 8 Juniors, 14 Sophomores, 4 Others.

Source: Harvard University. Annual Reports of the President and Treasurer of Harvard College, 1900-01, p. 64.

1900-01
ECONOMICS 3.
[Mid-year Examination]

Answer only ten questions.

  1. Upon what does Kidd base his argument that religion is necessary to keep men from taking such political action as would suspend economic competition, and what is the crucial point in his argument?
  2. In the light of Kidd’s theory of social evolution, discuss the question: Can there be a permanent civilization? Or, do the conditions which promote progress also ensure decay?
  3. Classify the sanctions for conduct which originate outside the individual and explain your classification.
  4. Explain and illustrate the meaning of the following: “Generalizing this struggle and extending it to every form existing in the social life — linguistic, religious, political, artistic, and moral, as well as industrial — we see that the really fundamental social opposition must be sought for in the bosom of the social individual himself.” (Tarde, Social Laws. Ch. II. p. 83.)
  5. What is meant by social stratification? How does it originate? What are some of its consequences?
  6. Compare Herbert Spencer’s theory of progress with Lester F. Ward’s, giving special attention to the argument which each offers in support of his theory.
  7. What, according to Patten, are the chief obstacles to a progressive evolution.
  8. Explain the following. “The difference between that society of conscious units which we call mind, and a society of human beings on our planet, is in the completeness of the mechanism.” (Patten, Theory of Social Forces. Ch. II. p. 21.)
  9. What, according to Patten, is the significance of the transition from a pain to a pleasure economy.
  10. How does Bagehot account for the origin of national traits?
  11. Discuss the question: Does charity retard the process of race improvement?

Source: Harvard University Archives. Harvard University Mid-year Examinations, 1852-1943. Box 4, Bound volume: Examination Papers, Mid-Years, 1900-01.

1900-01
ECONOMICS 3.
[Year-end Examination]

Discuss the following topics

  1. The definition of progress.
  2. Charity as a factor in human selection.
  3. The way in which, according to Spencer, the different classes of institutions are related to one another.
  4. The sanctions for conduct.
  5. A moral ideal as a factor in human selection.
  6. The natural antagonism of human interests and the problem of evil.
  7. The storing of the surplus energy of society.
  8. The influence of property on the relations of the sexes.
  9. Labor and service as bases of distributive justice
  10. The influence of militarism upon race development, or deterioration.

Source: Harvard University Archives. Harvard University, Examination Papers, 1873-1915. Box 5, Bound volume: Examination Papers, 1900-01. Papers Set for Final Examinations in History, Government, Economics, Philosophy, Education, Fine Arts, Architecture, Landscape Design, Music in Harvard College (June, 1901), p. 24.

 

__________________________________

Economics 5.
Railways and Other Public Works

For Undergraduates and Graduates.

Course Announcements

51 hf. Railways and Other Public Works, under Public and Corporate Management. Half-course (first half-year). Tu., Th. and (at the pleasure of the instructor) Sat., at 1.30. Mr. Meyer.

52 hf. Railways and Other Public Works (advanced course). Half-course (second half-year). Tu., Th. and (at the pleasure of the instructor) Sat., at 1.30. Mr. Meyer.

Source: Harvard University. Announcement of the Courses of Instruction provided by the Faculty of Arts and Sciences for the Academic Year 1900-1901, p. 42.

Enrollments

[Economics] 51 hf. Mr. [Hugo Richard] Meyer.— Railways and other Public Works, under Public and Corporate Management.

Total 86: 4 Graduates, 52 Seniors, 17 Juniors, 4 Sophomores, 9 Others.

[Economics] 52 hf. Mr. Meyer.— Railways and other Public Works (advanced course).

Total 9: 3 Graduates, 4 Seniors, 1 Junior, 1 Sophomore.

Source: Harvard University. Annual Report of the President of Harvard College, 1900-1901, p.64.

1900-01
ECONOMICS 51
[Mid-year Examination]

Omit the last question if the paper seems too long

  1. The construction put upon the long and short haul clause: by the Interstate Commerce Commission; by the Supreme Court.
  2. The decisions of the Interstate Commerce Commission on group rates.
  3. The railway rate situation in Germany [Prussia]; does it throw any light on the railway problem in the United States?
  4. “If pooling produces any beneficial result, it necessarily does so at the expense of competition. It is only by destroying competition that the inducement to deviate from the published rate is wholly removed….By the legalizing of pooling the public loses the only protection which it now has against the unreasonable exactions of transportation agencies.”—Give your reasons for accepting or rejecting this statement.
    Alternative:—
    The reasons for the instability of pools in the United States.
  5. The Iowa Railroad Commission.
    Alternative:—
    To what extent was the long and short haul clause of the Interstate Commerce Act enforced; what was the effect of that enforcement: on railway revenues; on intermediate shipping or distributing points?
  6. The body of administrative law to be found in the decisions of the Massachusetts Gas and Electric Light Commission’s decisions upon petitions for reductions in the price of gas.
  7. (a) Is it to the public interest to insert in street railway charters provisions seeking to secure to the municipality or the state a share in any excess of profit over the normal rate?
    Alternative: (b) and (c).
    (b) The evidence as to the return on capital obtainable in street railway ventures.
    (c) What questions of public policy were raised in the case of the Milwaukee Street Railway and Electric Light Co. vs. The City of Milwaukee?
  8. What statistics were used in illustrating in a general way the statement that railway charges are based upon what the traffic will bear; in discussing the bearing of stock-watering upon railway rates; in discussing the return obtained by capital invested in railway enterprises in the United States?

Source: Harvard University Archives. Examination Papers, 1873-1915. Box 5. Bound Volume: Examination Papers 1900-01. Papers Set for Final Examinations in History, Government, Economics, Philosophy, Education, Fine Arts, Architecture, Landscape Design, Music in Harvard CollegeJune, Pages 24-25.

1900-01
ECONOMICS 52
[Year-end Examination]
  1. The railways and the national finances in Prussia and Australia.
  2. Railway rates and the export trade of the United States since 1893, or, 1896.
  3. The economic situation in Australia since 1892, and the Australian railways.
  4. “A fatal objection to the income or preference bond is that it is an attempt to combine two contradictory commercial principles.”
    Discuss this statement fully. What does it mean? Is it true?
  5. If you had access to all the accounts of a railroad, how should you determine the value to it of one of its branch lines?
  6. To what accounts would you charge the following expenditures? (If you do not remember the exact Interstate Commerce Commission classification, use your best judgment.) State reasons in each case.
    Engineer’s wages on a special train conveying the general manager to an extensive flood covering the line.
    Fireman’s wages on an engine employed exclusively in switching to and from the repair shops.
    Conductor’s wages on a worktrain engaged in taking up rails on an abandoned branch.
    Brakeman’s wages on a train engaged solely in hauling company’s coal for company’s use.
    Cost of taking up comparatively new sound rails judged too light for heavy rolling stock.
    Cost at a competitive point of a new station to replace an old one which was large enough but old-fashioned.
  7. State the commonest problems facing a reorganization committee for an insolvent road, and then suggest and defend one course of procedure for each problem.
  8. Combine and arrange the following items so as to give the best information about the operation and condition of the road. (Do not rewrite the names but use the corresponding numbers where possible.)
1. Passenger train miles 2,000,000
2. Freight train miles 3,400,000
3. Passenger train earnings $2,400,000
4. Freight train earnings $5,500,000
5. Income from investments $100,000
6. Dividends $500,000
7. Operating expenses $4,700,000
8. Av. no. pass. cars per train 4
9. Av. no. passengers per car 11
10. Tons freight carried 2,800,000
11. Av. load per car (loaded and empty), tons 8.2
12. Av. no. loaded cars per train 12.3
13. Av. no. empty cars per train 6.7
14. Interest charge for year $2,200,000
15. Due other roads $100,000
16. Stocks and bonds owned $4,900,000
17. Supplies on hand $500,000
18. Taxes for the year $300,000
19. Accounts receivable $500,000
20. Cash $1,000,000
21. Surplus for the year $300,000
22. Profit and loss account $1,000,000
23. Taxes accrued but not due $100,000
24. Capital stock $50,000,000
25. Interest due $700,000
26. Funded debt $45,000,000
27. Due from other roads $100,000
28. Interest accrued not due $300,000
29. Franchises and property $90,400,000
30. Bonds of the company in its treasury $800,000
31. Accounts payable $1,000,000
32. No. of passengers carried 2,300,000

Source: Harvard University Archives. Examination Papers, 1873-1915. Box 5. Bound Volume: Examination Papers 1900-01. Papers Set for Final Examinations in History, Government, Economics, Philosophy, Education, Fine Arts, Architecture, Landscape Design, Music in Harvard College. June, 1901. Pages 25-27.

 

__________________________________

Economics 6.
Economic History of the U.S.

For Undergraduates and Graduates.

Course Announcement
  1. The Economic History of the United States. Tu., Th., at 2.30, and a third hour at the pleasure of the instructor. Mr. —.

Source: Harvard University. Announcement of the Courses of Instruction provided by the Faculty of Arts and Sciences for the Academic Year 1900-1901, p. 42.

Enrollment
  1. Professor Taussig. — The Economic History of the United States.

Total 164: 9 Graduates, 63 Seniors, 68 Juniors, 13 Sophomores, 11 Others.

Source: Harvard University. Annual Reports of the President and Treasurer of Harvard College, 1900-01, p. 64.

1900-01
ECONOMICS 6
[Mid-year Examination]

Arrange your answers strictly in the order of the questions. Answer all the questions,

  1. The nature and object of the scales of depreciation established by Congress and by the States at the close of the war of the Revolution; and how far these objects were accomplished.
  2. “The year 1789 marks no such epoch in economies as it does in political history.” — Taussig. How far is this true as to (1) financial legislation; (2) tariff legislation; (3) the course of foreign trade; (4) the growth of manufactures?
  3. Explain how you would distinguish Treasury Notes designed to circulate as currency from those designed simply to meet financial needs; and state when and under what circumstances, between 1789 and 1860, the United States resorted to issues of the first kind.
  4. Suppose the charter of the first Bank of the United States had been renewed: would the effect have been favorable or unfavorable for the finances of the government, for the bank, for the community, in 1812-1815?
  5. Suppose the charter of the second Bank of the United States had been renewed: would the effect have been favorable or unfavorable for the finances of the government, for the bank, for the community, in 1835-40?
  6. Describe the Independent Treasury system, as first established and as finally settled (give dates). Do you believe it better than the alternative system proposed by its opponents? Why?
  7. The causes of the crises of 1837 and 1857: wherein similar, wherein different.
  8. State what were the duties on cotton goods in 1809, 1819, 1839, 1859; and give your opinion whether the duties at these several dates were designed to give protection, and whether protection was then expedient.
  9. Why the early railway enterprises of the States were undertaken as public enterprises; and how far their history may be fairly cited for or against the policy of public management.

Source: Harvard University Archives. Harvard University Mid-year Examinations, 1852-1943. Box 4, Bound volume: Examination Papers, Mid-Years, 1900-01.

1900-01
ECONOMICS 6
[Year-end Examination]

Arrange your answers strictly in the order of the questions

  1. Explain summarily at what dates and to what extent land-grants and bond-subsidies were extended to railways by the United States; and state whether you believe these measures brought advantage to the country.
  2. Was the management of the finances during the Civil War fraught with more or less evil consequences than that during the War of 1812, as regards (1) the currency, (2) the banks?
  3. State what main sources of revenue were expected to be used, what were used in fact, by the United States in each of the years 1862, 1863, 1864; and explain how the resort to the sources actually used came about.
  4. For the decade 1870-80, explain the connection between the course of prices, foreign trade, railway operations, and currency legislation.
  5. For the decade 1880-90, connect the history of the public debt, the national revenues, the banking system, the silver currency.
  6. Does the argument for protection to young industries find support in the history of (1) the cotton manufacture before 1830, (2) the silk manufacture since 1870, (3) the tin plate industry since 1890.
  7. Explain how the theory of comparative costs may be applicable to the present situation as regards carpet wool, beet sugar, glassware, woollen cloths (take three).
  8. What changes were made in the duties on raw and refined sugar in 1890, 1894, 1897? Which mode of treatment do you regard the most advisable, and why?
  9. State what causes you believe to have chiefly promoted the growth and maintenance of the sugar and oil combinations; and consider which of these two you regard as typical, and as instructive for forecasting the future of combinations.

Source: Harvard University Archives. Examination Papers, 1873-1915. Box 5. Bound Volume: Examination Papers 1900-01. Papers Set for Final Examinations in History, Government, Economics, Philosophy, Education, Fine Arts, Architecture, Landscape Design, Music in Harvard College. June, 1901. Pages 27-28.

 

__________________________________

Economics 81
Money

For Undergraduates and Graduates.

Course Announcement

81 hf. Money. Half-course (first half-year). Tu., Th., Sat., at 11. Mr. Andrew.

Source: Harvard University. Announcement of the Courses of Instruction provided by the Faculty of Arts and Sciences for the Academic Year 1900-1901, p. 42.

Enrollment

81 hf. Dr. [Abram Piatt] Andrew. Money.

Total 122: 3 Graduates, 56 Seniors, 41 Juniors, 8 Sophomores, 1 Freshman, 13 Others.

Source: Harvard University. Annual Reports of the President and Treasurer of Harvard College, 1900-01, p. 64.

1900-01
ECONOMICS 8
[Mid-year Examination]

Answer only three questions from each group, but consider the questions strictly in the order of their arrangement 

I

  1. What is meant by

(1) a “double” standard;
(2) a “parallel” standard;
(3) a “limping” standard;
(4) a “single” standard;

Cite at least two historic examples of each, giving approximate dates.

  1. The following are estimates which have been made of the average production of silver, and its annual average export to the Orient in millions of ounces:

Production Export to East
1851-55 28 mill.

20 mill.

1855-60

29  ” 52  ”
1861-65 35  ”

53  ”

1865-70

43  ”

25  ”

State the causes of the singular situation revealed in these figures, and explain its actual effect upon the relative values of gold and silver in Europe.

  1. Suppose that the British government in 1870 had used the right conferred by the act of 1816, and had proclaimed the free coinage of silver at the ratio then current. What differences do you think would have occurred in the subsequent currency history of the world?
  2. Describe the effect of the suspension of the coinage of silver upon the value of the currency in each of the following cases:—
    (1) in Holland; (2) in Austria; (3) in Russia; (4) in India.

II

  1. “Before 1873 we had coined in the United States only about eight million silver dollars ($8,031,238) while since the date fixed as the beginning of demonetization we have coined nearly five hundred millions ($485,427,703).”
    How do you explain (1) the small amount of dollars coined before 1873? (2) the large amount coined since then?
  2. What in your opinion was the real significance of (1) the act of 1803? (2) the act of 1873?
  3. “With the exception of the brief period of fifteen years (1544-60) the English coins have never been debased.”
    In what sense and to what extent is this statement correct?
  4. In writing of the currency history of England during the years, immediately succeeding the great recoinage (1696) Mr. Dana Horton says:—
    “And so the full weight standard coin of the Realm, to create a stock of which the State had spent a sum greater than its regular annual revenue, and equal to perhaps a fourth of the country’s total stock of cash, — was allowed to find its way back to the melting-pot in exchange for cheaper gold.”
    Explain the situation to which he refers, and the reasons for this disappearance of the “standard coin.”

III

  1. (a) What were the main arguments which Lord Liverpool advanced in favor of a single gold standard?
    (b) What were the legislative acts in which his influence is to be traced?
  2. (a) Do falling prices necessarily mean an increase in the burden of debts?
    (b) Do they in the long run inevitably diminish the productiveness of industry?
    (c) To what extent are they prejudicial to the interests of the working classes?
  3. “It is generally agreed that every fall in the value of silver acted at the time as a stimulus to Indian exports and as a check on imports into India.”
    (1) Explain this statement, (2) state how far it is confirmed by commercial statistics, and (3) show whether such a condition is ever likely to be of prolonged duration.
  4. It is alleged that the Russian government, by stimulating exports, and hindering imports, has endeavored to secure a favorable balance of trade, with the idea of increasing the quantity of gold in the country? What do you think would be the ultimate effect of such a policy if continuously pursued?

Source: Harvard University Archives. Harvard University Mid-year Examinations, 1852-1943. Box 4, Bound volume: Examination Papers, Mid-Years, 1900-01.

Also: Harvard University Archives. Examination Papers, 1873-1915. Box 5. Bound Volume: Examination Papers 1900-01. Papers Set for Final Examinations in History, Government, Economics, Philosophy, Education, Fine Arts, Architecture, Landscape Design, Music in Harvard College (June, 1901), pp. 28-30.

 

__________________________________

Economics 9.
The Labor Question in Europe and the U.S.

For Undergraduates and Graduates.

Taught by W. F. Willoughby (Edward Cummings’ successor).

Course Announcement
  1. The Labor Question in Europe and the United States. — The Social and Economic Condition of Workingmen. Tu., Th., Sat., at 10. Mr. —.

Source: Harvard University. Announcement of the Courses of Instruction provided by the Faculty of Arts and Sciences for the Academic Year 1900-1901, p. 42.

Enrollment

92 hf. Mr. W. F. Willoughby. — The Labor Question in Europe and the United States. The Social and Economic Condition of Workingmen.

Total 146: 3 Graduates, 53 Seniors, 40 Juniors, 35 Sophomores, 3 Freshmen, 12 Others.

Source: Harvard University. Annual Reports of the President and Treasurer of Harvard College, 1900-01, p. 64.

1900-01
ECONOMICS 9
[Year-end examination]
  1. Show how the change in the organization of industry from the handicraft system and production on a small scale to the factory system and production on a large scale has led to; (a) efforts to supplant the wages system by socialism[,] coöperation, etc., (b) the trade union movement, and (c) compulsory compensation acts.
  2. Give the arguments for and against profit-sharing as regards (a) it being a more just system of enumeration than the wages system, and (b) its practical advantages.
  3. What are the two systems of coöperative production now practice in Great Britain, and why are they meeting with more success than earlier efforts?
  4. Describe the trade agreement between the National Metal Trades Association and the International Association of Machinists in such a way as to show its essential character and significance, and particularly its relation to the trade union movement and the question of the prevention and adjustment of industrial disputes.
  5. What was the nature of the “new unionism” movement in Great Britain, and its success?
  6. What is the general character of the Massachusetts State Board of Conciliation and Arbitration; what its duties and its powers?
  7. Describe the essential features of the French Workmen’s Compensation Act.
  8. Give a brief sketch of the Social Democratic Party in Germany, with the names of its early leaders and important events in its history.
  9. In what ways can the municipality take action for the improvement of the housing condition of the poorer classes without itself building tenements? What are some of the objections to the municipalities themselves undertaking building operations?
  10. Show why employment bureaus can do but little for the solution of the general problem of the unemployed.

Source: Harvard University Archives. Examination Papers, 1873-1915. Box 5. Bound Volume: Examination Papers 1900-01. Papers Set for Final Examinations in History, Government, Economics, Philosophy, Education, Fine Arts, Architecture, Landscape Design, Music in Harvard College (June, 1901), pp. 30-31.

Enrollment (Economics 9a)

9a2 hf. Mr. W. F. Willoughby. — Provident Institutions. Workingmen’s Insurance, Friendly Societies, Savings Banks.

Total 22: 1 Graduate, 13 Seniors, 5 Juniors, 2 Sophomores, 1 Other.

Source: Harvard University. Annual Reports of the President and Treasurer of Harvard College, 1900-01, p. 64.

1900-1901
ECONOMICS 9a
[Year-end Examination]
  1. What is the general situation in France at the present time in respect to insurance against old age and invalidity? Describe briefly the organization and workings of important institutions, and show particularly how the government is attempting to further this kind of insurance.
  2. What has been the general policy of the British government in respect to the regulation of Friendly Societies? Give the main features of law now regulating them.
  3. Describe the Fraternal Beneficial Orders of the United States as regards (a) their general scheme of organization, and (b) system of insurance.
  4. Show wherein this insurance system is defective by contrasting it with that of ordinary life insurance companies: indicate reforms that are necessary and how they can best be brought about.
  5. Contrast the systems of savings banks in Great Britain and the United States.
  6. In what respects are coöperative banks of the Schulze-Delitzsch and Raiffeisen type more valuable social institutions than the ordinary savings banks?
  7. Describe the principles upon which all coöperative building and loan associations in this country are organized, and indicate ways in which they might profitably be subjected to more rigid state control.
  8. Why is it impracticable to insure against unemployment?
  9. Outline briefly the system of sick insurance in Germany.

Source: Harvard University Archives. Examination Papers, 1873-1915. Box 5. Bound Volume: Examination Papers 1900-01. Papers Set for Final Examinations in History, Government, Economics, Philosophy, Education, Fine Arts, Architecture, Landscape Design, Music in Harvard College (June, 1901), p. 31.

 

__________________________________

Economics 10.
Mediaeval Economic History of Europe.

For Undergraduates and Graduates.

Course Announcement

For Undergraduates and Graduates.

  1. The Mediaeval Economic History of Europe. Tu., Th., and (at the pleasure of the instructor) Sat., at 12. Professor Ashley.

Source: Harvard University. Announcement of the Courses of Instruction provided by the Faculty of Arts and Sciences for the Academic Year 1900-1901, p. 41.

Enrollment
  1. Professor Ashley. The Mediaeval Economic History of Europe.

Total 11: 6 Graduates, 4 Seniors, 1 Junior.

Source: Harvard University. Annual Reports of the President and Treasurer of Harvard College, 1900-01, p. 64.

1900-01
ECONOMICS 10
[Mid-year Examination]

Not more than six questions should be attempted, of which the first should be one.

  1. Translate, and briefly comment upon
    1. Toto regis Willelmi primi tempore perseveravit haec institutio, usque tempora regis Henrici filii ejus; adeo ut viderim ego ipse quosdam qui victualia statutis temporibus de fundis regiis ad curiam deferri viderint.
    2. In Kateringes sunt X hidae ad geldum Regis. Et de istis X hidis tenent XL villani XL virgas terrae.
    3. Compotus Roberti Oldeman praepositi de Cuxham, ab in crastino Sancti Jacobi anno regni Regis Edwardi filii Regis Edwardi decimo usque ad in crastinum Sancti Jacobi proxime sequentis anno regni Regis Edwardi praedicti undecimo intrante.
    4. Rogamus . . . ademptum sit jus etiam procuratoribus nedum conductori adversus colons ampliandi partes agrarias aut operarum praebitionem jugorumve.
    5. Orgeterix ad judicium omnem suam familiam, ad hominum milia decem, undeque coëgit et omnes clientes obaeratoque suos quorum magnum numerum habebat eodem conduxit.
  2. What materials have we for forming a judgment as to the position of the rural population of England in the period from the eleventh to the fourteenth centuries? Classify them, and indicate the value of each class for the purposes of this enquiry.
  3. Wherein did the status of the coloni of the later Roman Empire resemble or differ from that of the medieval villein?
  4. Describe the constitution and working of manorial courts. What light does their history throw on the evolution of social classes?
  5. “Wie das Wort Dorf … dem Sinne nach einen Haufen bezeichnet, so ist auch haufenförmig oder Haufendorf der geeignetste Ausdruck für diese Art der Dorfenlage.” Explain and comment.
  6. “M. Fustel took for his point of departure the Provincial villa; Dr. Hildebrand takes the Kirghises of modern Asia.” Explain, and then show the peculiar dangers of each method.
  7. “We may safely follow Palgrave in taking the Anglo-Saxon townships as the integral molecules out of which the Anglo-Saxon State was formed.” Why? or why not?
  8. What was the gwely? What bearing has it on the general problem of “tribal” organization?
  9. What are the assumptions or postulates of modern Political Economy? To what extent were they true of the Middle Ages?
  10. Which book read in connection with this course has interested you most? Describe its method and estimate the value of its contribution to economic history.

Source: Harvard University Archives. Harvard University Mid-year Examinations, 1852-1943. Box 4, Bound volume: Examination Papers, Mid-Years, 1900-01.

1900-01
ECONOMICS 10
[Year-end Examination]

Not more than six questions should be attempted, of which the first should be one

  1. Briefly comment upon the following passages, and translate such of them as are not in English:—
    1. Colunt discreti ac diversi, ut fons, ut campus, ut nemus placuit. Vicos locant non in nostrum morem connexis et cohaerentibus aedificiis: suam quisque domum spatio circumdat.
    2. If a man agree for a yard of land or more, at a fixed rent, and plough it; if the lord desire to raise the land to him to service and to rent, he need not take it upon him, if the lord do not give him a dwelling.
    3. Ego S. … et ego P. … aliquantulam agri partem pro remedio animarum nostrorum W. episcopo in dominio donare decrevimus; id est xxx cassatorum in loco qui dicitur T.
    4. Si quis super alterum in villa migrare voluerit, et unus vel aliqui de ipsis qui in villa consistunt eum suscipere voluerit, si vel unus extiterit qui contradicat, migrandi ibidem licentiam non habebit.
    5. Qui habebant de tenentibus per diaetas totius anni, ut assolet de nativis, oportebat eos relaxare et remittere talia opera.
    6. If any one does an injury who is not of the gild and is of the franchise … he shall lose his franchise.
  2. Explain the position of Maitland’s Domesday Book and Beyond in the discussion concerning the origin of the manor.
  3. Distinguish between the several characteristics of mediaeval towns, and indicate the part played by each, in your opinion, in the formation of specifically urban conditions.
  4. Examine the relations between questions of personal status and questions of economic condition in relation to the ‘peasants’ of the Middle Ages.
  5. What is the nature of our evidence as to the Peasants’ Rising of 1381? Is there any reason for ascribing anything like an economic programme to the leaders of the movement?
  6. Indicate briefly (a), the several influences tending towards a corporate organization of industry in the later Middle Ages and (b) the advantages or disadvantages of such an organization.
  7. Distinguish between the several immigrations of foreign work people to England before the accession of James I, and explain the nature of their contributions to the development of English manufactures.
  8. The relation of John Major and Juan Vives to the development of the English ‘Poor Law.’
  9. What changes, if any, did the Reformation bring about in social life?

Source: Harvard University Archives. Examination Papers, 1873-1915. Box 5. Bound Volume: Examination Papers 1900-01. Papers Set for Final Examinations in History, Government, Economics, Philosophy, Education, Fine Arts, Architecture, Landscape Design, Music in Harvard College (June, 1901), pp. 32-33.

 

__________________________________

Economics 122.
Banking and the History of the Leading Banking Systems.

For Undergraduates and Graduates.

Course Announcement

122 hf. Banking and the history of the leading Banking Systems. Half-course (second half-year). Tu., Th., Sat., at 11. Dr. Sprague.

Source: Harvard University. Announcement of the Courses of Instruction provided by the Faculty of Arts and Sciences for the Academic Year 1900-1901, p. 43.

Enrollment

122 hf. Dr. [Oliver Mitchell Wentworth] Sprague. — Banking and the History of the Leading Banking Systems.

Total 128: 4 Graduates, 51 Seniors, 43 Juniors, 16 Sophomores, 14 Others.

Source: Harvard University. Report of the President of Harvard College, 1900-1901, p. 64.

1900-01
ECONOMICS 122
[Year-end Examination]

Arrange your answers strictly in the order of the questions. Answer all the questions under A and two of those under B

A

  1. Explain in detail and under different circumstances the effect of an advance of the rate of discount by the Bank of England upon the money market of London and upon the foreign exchanges.
  2. Taking the separate items of a bank account point out how those of the Bank of Amsterdam differed from those of a modern bank.
  3. Define and explain:—
    1. Bill broker.
    2. Banking Principle.
    3. The State Bank of Indiana.
    4. The banking law of Louisiana.
    5. Clearing House Certificates.
  4. The extent and banking consequences of government control of the Bank of France and the Reichsbank.
  5. How do government receipts and expenditures affect the money market (a) of London, (b) of New York?
  6. Explain with illustrations from the crises of 1857 and 1893 the nature of the demand for cash in time of crisis, and consider how far that demand may be met under a flexible system of note issue.

B

  1. (a) How far and with what qualifications may banking experience in the United States before 1860 be appealed to in the discussion of changes in our banking system? (b) How far, similarly, may Canadian experience be applied?
  2. “Why compel banks to send home for redemption a multitude of notes which can as well be used in payments and are sure to be reissued at once? Why impede the free use of its power of circulation by any enterprising bank by requiring the early redemption of notes which the holder does not in fact care or need to have redeemed?”
    Explain from past experience what regulations may be expected to bring about these results, and give the reasons for demanding them.
  3. Discuss the question of branch banking with reference to the United States, including in your discussion considerations of safety and economy. Would branch banking be more desirable than at present if notes were issued against general banking assets.

Source: Harvard University Archives. Examination Papers, 1873-1915. Box 5. Bound Volume: Examination Papers 1900-01. Papers Set for Final Examinations in History, Government, Economics, Philosophy, Education, Fine Arts, Architecture, Landscape Design, Music in Harvard College (June, 1901), pp. 34-35.

 

__________________________________

Economics 12a1.
International Payments and the Flow of Precious Metals.

For Undergraduates and Graduates.

Course Announcement

12a2 hf. International Payments and the Flow of the Precious Metals. Half-course (second half-year). Three times a week. Mr—.

Source: Harvard University. Announcement of the Courses of Instruction provided by the Faculty of Arts and Sciences for the Academic Year 1900-1901, p. 43.

Enrollment

[Economics] 12a1 . Mr. [Hugo Richard] Meyer.—International Payments and the Flow of the Precious Metals.

Total 16: 2 Graduates, 9 Seniors, 4 Juniors, 1 Other.

Source: Harvard University. Report of the President of Harvard College, 1900-1901, p. 64.

1900-01
ECONOMICS 12a1.
[Mid-Year Examination]

Observe strictly the order in which the questions are arranged.

  1. Sidgwick’s criticisms on Mill’s doctrine of international trade and their validity.
  2. What temporary changes in the general level of prices in this country should you expect to see, as the result of a large permanent withdrawal of foreign capital? What ultimate change of prices should you expect?
  3. Suppose the exportation of specie from the United States to be prohibited (or, as has sometimes been suggested, to be slightly hindered), what would be the effect on rates of exchange, and on prices of goods, either domestic or foreign? Would the country be a loser or not? [See Ricardo (McCulloch’s ed.), page 139.]
  4. The conditions which led to the flow of gold to the United States in the fiscal years 1880 and 1881?
  5. What economic conditions or events tended to make the year 1890 a turning point both in domestic and in international finance?

Alternative:

The reasons for the return flow from Europe of American securities in the years 1890-1900?

  1. What sort of wealth did France actually sacrifice in paying the indemnity? What was the process?
  2. Is Mr. Clare justified in making the general statement that “the gold-points mark the highest level to which an exchange may rise, and the lowest to which it may fall”?
  3. Why is it that certain trades bills are drawn chiefly, or even exclusively, in one direction, e.g. by New York on London and not vice versa; and how is this practice made to answer the purpose of settling payments which have to be made in one direction?

Alternative:

Why has England become the natural clearing-house for the world?

Source: Harvard University Archives. Harvard University Mid-year Examinations, 1852-1943. Box 4, Bound volume: Examination Papers, Mid-Years, 1900-01.

Also: Harvard University Archives. Examination Papers, 1873-1915. Box 5. Bound Volume: Examination Papers 1900-01. Papers Set for Final Examinations in History, Government, Economics, Philosophy, Education, Fine Arts, Architecture, Landscape Design, Music in Harvard College (June, 1901), pp. 33-34.

 

__________________________________

Economics 13.
Methods of Economic Investigation.

Primarily for Graduates.

Course Announcement
  1. Methods of Economic Investigation.—English Writers. German Writers. Tu., Th., at 1.30. Professor Taussig.
    Courses 15 and 13 are usually given in alternate years.

[15. The History and Literature of Economics to the close of the Eighteenth Century. Mon., Wed., and (at the pleasure of the instructor) Fri., at 12. Professor Ashley.
Omitted in 1900-01.]

Source: Harvard University. Announcement of the Courses of Instruction provided by the Faculty of Arts and Sciences for the Academic Year 1900-1901, p. 43.

Enrollment
1900-01

Economics 132 hf. Asst. Professor Carver. — Methods of Economic Investigation.

Total 10: 4 Graduates, 6 Seniors.

Source: Harvard University. Report of the President of Harvard College, 1900-1901, p. 64.

1900-01
ECONOMICS 13
[Year-end Examination]

Discuss ten of the following topics.

  1. The subdivision of economics into departments.
  2. The fields for the observation of economic phenomena.
  3. The place of historical and statistical research in economic investigation.

4, 5, 6. The methods of investigating:

    1. The causes of poverty.
    2. The effect of immigration on the total population of the United States.
    3. The effect of protection on the production of flax fibre, on the iron industry, or on any other industry which you may select.
  1. The nature of an economic law.
  2. The relation of the theory of probabilities to economic reasoning.
  3. The use of hypotheses in economic reasoning.
  4. The use of the terms “static” and dynamic in economic discussion.
  5. The use of diagrams and mathematical formulae in economic discussion.

Source: Harvard University Archives. Examination Papers, 1873-1915. Box 5. Bound Volume: Examination Papers 1900-01. Papers Set for Final Examinations in History, Government, Economics, Philosophy, Education, Fine Arts, Architecture, Landscape Design, Music in Harvard College (June, 1901), p. 35.

 

__________________________________

Economics 17.
Economic Organization and Resources of European Countries.

For Undergraduates and Graduates.

Course Announcement
  1. The Economic Organization and Resources of European countries. Mon., Wed., and (at the pleasure of the instructor) Fri., at 12. Professor Ashley.

Source: Harvard University. Announcement of the Courses of Instruction provided by the Faculty of Arts and Sciences for the Academic Year 1900-1901, p. 42.

Enrollment
  1. Professor Ashley. The Economic Organization and Resources of European countries.

Total 34: 5 Graduates, 14 Seniors, 9 Juniors, 3 Sophomores, 3 Others.

Source: Harvard University. Annual Reports of the President and Treasurer of Harvard College, 1900-01, p. 64.

1900-01
ECONOMICS 17
[Mid-year Examination]

Not more than eight questions should be attempted

  1. “It is less important for a particular community than ever it was to be in possession of cheap food and raw materials produced within its own domain.” Discuss this proposition.
  2. Describe very briefly the main features of the physical geography of England (illustrating your answer, if possible, with a map) and indicate in general terms their economic consequences.
  3. Set forth some of the general considerations which should be taken into account in answering the question whether the industrial development of Ireland has been injuriously affected by English legislation.
  4. Compare the number and character of the several classes maintained by agriculture in England, with those of the agricultural classes in the U.S. and on the continent of Europe.
  5. Explain the powers of dealing with his estate enjoyed at present by an English tenant for life under a settlement.
  6. What districts of England are now suffering most severely from agricultural depression, and why?
  7. Can any lessons be drawn for the U.S. from the recent history of productive coöperation in England? Give your reasons.
  8. Give a rapid survey of the apparent coal resources of the world.
  9. What points of especial interest are there to the economist in the history, situation, character, etc. of the South Wales Coal Field?
  10. What is meant by Collective Bargaining? What are its prerequisites? What examples of it are you acquainted with in America?

Source: Harvard University Archives. Harvard University Mid-year Examinations, 1852-1943. Box 4, Bound volume: Examination Papers, Mid-Years, 1900-01.

1900-01
ECONOMICS 17
[Year-end Examination]

Not more than eight questions should be attempted

  1. The British Chancellor of the Exchequer proposes to levy a duty of one shilling per ton upon the export of coal from the United Kingdom: He argues that the tax will not be borne by the producer, but mainly, if not wholly, by the foreign consumer. Consider (a) what are the conditions under which this is likely to be the case, (b) how far these conditions are at present realized in the case of England.
  2. Distinguish the successive stages in the technological history of iron and steel, and connect them with the industrial development of the several countries concerned.
  3. What were the questions at issue in England in the Engineering dispute of 1897? What, with your present knowledge, do you think ought to have been your attitude, had you then been (a) an English engineering employer, (b) a leading official of the employees’ union.
  4. Give a brief account of the organization of the English cotton manufacture (as distinguished from the securing either of the material or of a market for the product). Contrast it with American conditions; and consider how England and New England are likely to be affected by the growth of the manufacture in the Southern States.
  5. Distinguish between the several forms of capitalist combination at present to be observed in England. What general causes have led to the movement? What, if any, advantages does it promise, and what, if any, dangers does it threaten?
  6. Compare Bradford and Roubaix in any aspects which seem to you worthy of attention.
  7. “Lorsque il n’y a point d’hommes riches qui aient de gros capitaux à mettre dans les entreprises d’agriculture, lorsque les récoltes ne suffisent pas pour assurer aut entrepreneurs des profits égaux à ceux qu’ils tireraient de leur argent en l’employant de toute autre manière, on ne trouve point de fermiers qui veuilient louer les terres. Les propriétaires sont forcées de les faire cultiver par les métayers hors d’état de faire aucunes avances et de bien cultiver. Le propriétaire fait lui-même des avances médiocres qui lui produisent un très médiocre revenu.”
    Translate the passage from Turgot; and then consider how far his description applies to existing conditions in France and Italy.
  8. Show the relation of the great manufacturing industries of France to the distribution of coal in that country.
  9. Would the construction of the Rhine-Elbe canal be a benefit to Germany? Give your reasons.
  10. “Wir müssen uns Rechenschaft ablegen, ob ohne eine grössere Macht zur See, ohne eine solche die unsere Küsten vor Blockaden schützt, unseren Kolonialbesitz und unseren Welthandel absolut sicher stellt, unsere wirtschaftliche Zukunft gesichert sei.”
    Are there sufficient reasons in the contemporary situation of Germany for this anxiety on the part of Professor Schmoller?
  11. (a) Give a brief account of the contents, and then (b) compare the method and general attitude toward the subject, of von Schulze-Gaevernitz’s Social Peace and de Rousers’ Labour Question in Britain.
  12. What in the light of the experience in the English coal, iron, and cotton industries, would seem to you the most satisfactory form to be taken by joint wage agreements in the great industries of America?

Source: Harvard University Archives. Examination Papers, 1873-1915. Box 5. Bound Volume: Examination Papers 1900-01. Papers Set for Final Examinations in History, Government, Economics, Philosophy, Education, Fine Arts, Architecture, Landscape Design, Music in Harvard College (June, 1901), pp. 36-37.

 

__________________________________

Economics 18.
Principles of Accounting.

For Undergraduates and Graduates.

Course Announcement

181 hf. The Principles of Accounting. — Lectures, discussions, and reports. Half-course (first half-year). Mon., Wed., and (at the pleasure of the instructor) Fri., at 3.30. Mr. W. M. Cole.

Source: Harvard University. Announcement of the Courses of Instruction provided by the Faculty of Arts and Sciences for the Academic Year 1900-1901, p. 43.

Enrollment

181 hf. Mr. W. M. Cole. — The Principles of Accounting.

Total 56: 43 Seniors, 4 Juniors, 2 Sophomores, 7 Others.

Source: Harvard University. Annual Reports of the President and Treasurer of Harvard College, 1900-01, p. 64.

1900-01
ECONOMICS 18
[Mid-year Examination]

Problems 1 to 5 inclusive form a connected whole;
but 6
and 7 may be substituted for 4 and 5

I

  1. Construct a rough ledger (omitting rulings and index-memoranda) to correspond with the following trial-balance:
Real estate $150,000 Proprietor $244,275
Plant $60,000 Merchandise $401,000
Patents $40,000 Rent $6,000
Supplies $228,000 Bills payable $14,000
Wages $127,000 Accounts payable $43,000
Coal $9,000 Reserve fund $12,000
Insurance $4,500
Trade discounts $8,000
Interest $1,500
Bills receivable $10,000
Accounts receivable $68,000
Cash $14,275
$720,275 $720,275
  1. The above trial-balance is supposed to be taken from manufacturing books that are kept on the ordinary commercial plan, i.e., without distinctive accounts for stores, manufacturing, stock, or trading; and to construct such accounts now is supposed to be either impossible or undesirable.
    If you were required to determine profit and loss for the year which these figures cover, what questions about the business should you wish to ask before reaching your conclusions? [Give your answer in the form of questions consecutively numbered.]
  2. State what would be fairly reasonable answers to your own questions above numbering the answers to correspond with the questions; and then, assuming your answers to be the real answers show a complete statement of resources and liabilities and of profit and loss.
  3. Close for the year the ledger that you constructed indicating all balances that you have transferred to other accounts and all balances that you have carried down for the new year.
  4. From the ledger as it now stands draw off a balance sheet showing the condition of the business at the beginning of the new year, assuming that the loss or gain is carried directly to the proprietor’s account.
  5. Journalize the following:

A gives you his note for $100, bearing interest, dated a month ago.
You discount at a bank a note for $100 payable in a month
B gives you A’s note for $100 payable in one month, and buy goods for $100 on one month’s time.
Your book-keeper charged bills receivable and credited B when B paid his bill by your own note returned to you. A counter entry is to be made, so that the original wrong entry need not be erased

  1. What is the distinguishing feature of double entry? Are two postings made for every entry? If not, what devices are employed for reducing the number of postings?

II
Omit one

  1. The balance sheet of a corporation on January 1, 1899, stood as follows:
Real estate $50,000 Capital stock $200,000
Plant $95,000 Accounts payable $20,000
Horses, etc. $15,000 Bills payable $25,000
Patents $20,000 Profit and loss $15,000
Merchandise $30,000
Accounts receivable $30,000
Cash $20,000
$260,000 $260,000

On January 1, 1900, the books showed the following facts:

Real estate $55,000 Capital stock $200,000
Plant $88,000 Accounts payable $12,000
Horses, etc. $12,000 Bills payable $17,000
Patents $19,000 Profit and loss $33,000
Merchandise $42,000
Accounts receivable $28,000
Cash $18,000
$262,000 $262,000

What has become of the profits earned?

Should you recommend that a dividend be declared? State your reasons.

  1. How should you treat interest received on a bond bought above par?
  2. Describe the following, and state the distinguishing feature of each: a real account; a nominal account; a suspense account; reserve fund: a sinking fund
  3. If payments are received on account of goods in process of manufacture, should such payments appear on the balance sheet? If so, where?
  4. Describe three different methods of treating depreciation, and show how each would appear upon the books. To what circumstances on a railroad is each adapted?
  5. A corporation is formed to unite and continue the business of three concerns, A, B, and C, engaged in the same industry. The books of the concerns show the following:
A B C
Assets (valuation) $80,000 $160,000 $120,000
Liabilities (external) $20,000 $80,000 $90,000
Average profit, last three years 10% 14% 30%
Average profit, preceding three years 9 17 25
Average profit, prior three years 10 20 20

On what basis should you determine the total amount of capital stock to be issued by the new corporation, and on what basis should you apportion it to these three concerns?

Source: Harvard University Archives. Harvard University Mid-year Examinations, 1852-1943. Box 4, Bound volume: Examination Papers, Mid-Years, 1900-01.

 

__________________________________

Economics 19.
General View of Insurance.

Primarily for Graduates.

Course Announcement

192 hf. A General View of Insurance. — Lectures and reports. Half-course (second half-year). Mon., Wed., and (at the pleasure of the instructor) Fri., at 3.30. Professor Wambaugh.
Course 19 cannot be counted towards the degree of A.B.

Source: Harvard University. Announcement of the Courses of Instruction provided by the Faculty of Arts and Sciences for the Academic Year 1900-1901, p. 43.

Enrollment

192 hf. Professor Wambaugh. — A General View of Insurance.

Total 9: 6 Seniors, 3 Others.

Source: Harvard University. Annual Reports of the President and Treasurer of Harvard College, 1900-01, p. 64.

1900-01
ECONOMICS 192
[Year-end Examination]

One of the paragraphs may be omitted.

  1. From the point of view of the person procuring the policy, what is the purpose of insurance?
  2. From the point of view of the community, what are the advantages and the disadvantages of insurance?
  3. Give some account of three insurance books, pamphlets, or periodicals.
  4. Tell what you know of the history of insurance.
  5. Give a classification of the provision of the New York standard form of fire insurance policy,
  6. If either party to the fire insurance contract wishes to terminate the insurance, what are his rights?
  7. What are the benefits and the dangers of fire insurance by government?
  8. Describe ordinary life policies, single payment life policies, twenty payment life policies, endowment policies, tontine policies, assessment insurance.
  9. If a person thirty years of age wishes to obtain a life insurance policy for a single premium, how is the premium calculated?
  10. What are the chief differences between fire insurance and marine insurance?
  11. Discuss any insurance topic of which you have made a special study. 

Source: Harvard University Archives. Examination Papers, 1873-1915. Box 5. Bound Volume: Examination Papers 1900-01. Papers Set for Final Examinations in History, Government, Economics, Philosophy, Education, Fine Arts, Architecture, Landscape Design, Music in Harvard College (June, 1901), p. 40.

 

__________________________________

Economics 20d.
Adam Smith and Ricardo.

Primarily for Graduates.

Course Announcement

20d. Adam Smith and Ricardo. Half-course. Professor Taussig.

Source: Harvard University. Announcement of the Courses of Instruction provided by the Faculty of Arts and Sciences for the Academic Year 1900-1901, p. 43.

Enrollment

20d1 hf. Professor Taussig. — Adam Smith and Ricardo.

Total 12: 7 Graduates, 5 Seniors.

Source: Harvard University. Annual Reports of the President and Treasurer of Harvard College, 1900-01, p. 64.

1900-01
ECONOMICS 20d
[Final examination]
  1. Compare Ricardo’s conclusions with Adam Smith’s on the course of wages, profits, and rent, as society advances: discussing not only the conclusions themselves, but the reasoning by which the two writers arrive at them.
  2. Under what circumstances are real wages high, according to Adam Smith? according to Ricardo?
  3. Adam Smith’s doctrine on labor as a measure of value; Ricardo’s strictures thereon; and Ricardo’s own doctrine.
  4. S. Mill in his Autobiography says that “it was one of my father’s main objects to make me apply to Smith’s more superficial view of political economy the superior lights of Ricardo, and to detect what was fallacious in Smith’s arguments or erroneous in his conclusions.” Set forth how you believe the two Mills (father and son) set about this task as to Adam Smith’s reasoning on the following topics:—
    1. the mode in which the payment of heavy foreign obligations is brought about by the exportation of goods, not by the outflow of specie;
    2. the distinction between that land which always affords rent, and that which sometimes does and sometimes does not;
    3. the effect of foreign trade in raising the general rate of profits in a country.
  5. “That able but wrong-headed man, David Ricardo; shunted the car of Economic science on to a wrong line, a line, however, on which it was further urged by his equally able and wrong-headed admirer, John Stuart Mill.” — W. S. Jevons.
    What grounds are there for assenting to this judgment? What grounds for dissenting from it?

Source: Harvard University Archives. Examination Papers, 1873-1915. Box 5. Bound Volume: Examination Papers 1900-01. Papers Set for Final Examinations in History, Government, Economics, Philosophy, Education, Fine Arts, Architecture, Landscape Design, Music in Harvard College (June, 1901), pp. 40-41.

Image Source: Detail from cover of the Harvard Class Album 1946.

Categories
Exam Questions Harvard

Harvard. Final Examinations in Economics courses, 1892-1893

The economic historian William J. Ashley joined the Harvard economics department in 1892-93, joining Professors Charles F. Dunbar and Frank W. Taussig and the instructors Edward Cummings and William M. Cole. This post gives us a complete set of semester examinations for all the economics courses offered at Harvard and, as extra bonus, exams for the Social Ethics course taught in the philosophy department by Francis G. Peabody.

_________________________

1892-93.
PHILOSOPHY 5. ETHICS OF THE SOCIAL QUESTIONS.

Enrollment

[Philosophy] 5. Professor F. G. PEABODY. — The Ethics of the Social Questions. — The questions of Charity, Divorce, the Indians, Temperance, and the various phases of the Labor Question, as problems of practical Ethics. — Lectures, essays, and practical observations. 2 hours.

Total 131: 5 Graduates, 60 Seniors, 25 Juniors, 6 Sophomores, 24 Divinity, 11 Others.

Source: Harvard University. Report of the President of Harvard College, 1892-1893, p. 63.

 

1892-93.
PHILOSOPHY 5.
[Mid-Year Examination]

  1. Explain and illustrate the “correlation” of the Social Questions and the doctrine of Social Energy.
  2. How does the history of ethical theory illustrate the philosophy of the Social Questions?
  3. Compare the principles of the English Poor-Law with the principles of the Elberfeld System.
  4. The plan, scope, and results of Mr. Charles Booth’s Study of East London.
  5. The character of the migration to London and its effect on
    1. social conditions in London (Charles Booth, I. 501, II. 444);
    2. the problem of municipal charity.
  6. How do the Germans deal with the problem of unemployed tramps?
  7. One year of General Booth’s Social Scheme, — its achievements and its possible limitations.
  8. Define the modern Labor Question and note its special characteristics.
  9. What does Carlyle mean by:
    Gospel of Mammonism? (Bk. III. ch. 2.)
    Gospel of Dilettantism? (Bk. III. ch. 3.)
    Captains of industry? (Bk. IV. ch. 4.)
    Plugson of Undershot? (Bk. III. ch. 10.)

What is his lesson drawn from:
Gurth, the thrall of Cedric? (Bk. III. ch. 13; Bk. IV. ch. 5.)

  1. Ruskin’s doctrine of wealth, of wages, and of exchange. How far, in your opinion, is Ruskin’s view of Political Economy justifiable?

Source: Harvard University Archives. Harvard University, Mid-year examinations, 1852-1943. Box 3, Bound Volume: Examination Papers, Mid-Year 1892-93.

 

 

1892-1893
PHILOSOPHY 5.
[Year-End Final Examination]

  1. State the Labor Question in terms of Ethics, arranging the various industrial propositions of the day in the order of their ethical sufficiency. Explain your arrangement.
  2. Why does the Anarchist find encouragement in the philosophy of Herbert Spencer?
  3. What is the philosophy of history which encourages the Socialist?
  4. The practical advantages which the Socialist anticipates under his programme, his reasons therefor, and your own judgment of their probability.
  5. The substitute for money proposed by Marx; with Schäffle’s criticism of the proposal. (Schäffle, pp. 77-90).
  6. The German system of insurance against old age and invalidism, — its plan, scope and difficulties.
  7. The Familistère at Guise and its lessons for socialism.
  8. Why has Coöperation gained so large a place in English industry and had such meagre success in the United States?
  9. The method of Profit-Sharing adopted in the Maison Leclaire, the secret of its success and the limits of its application in other cases. (Sedley Taylor, pp. 13-20).
  10. The special characteristics of the latest liquor Legislation proposed in Germany, in England and in the United States.

Source: Harvard University Archives. Harvard University, Examination Papers, 1873-1915. Box 4, Bound Volume: Examination Papers, 1893-95, “Papers Set for Final Examinations in Philosophy, History, Government and Law, Economics, Fine Arts, and Music in Harvard College, June 1893”.

_________________________

1892-93.
ECONOMICS 1. PRINCIPLES OF ECONOMICS

Enrollment

[Economics] 1. Professors [Frank W.] TAUSSIG and [William J.] ASHLEY, and Messrs. [Edward] CUMMINGS and [William M.] COLE. —

First half-year:

Mill’s Principles of Political Economy. 3 hours.

Second half-year:

Division A (Theoretical): Mill’s Principles of Political Economy. — Cairnes’s Leading Principles of Political Economy. 3 hours.

Division B (Descriptive): Labor and Capital, Coöperation. — Hadley’s Railroad Transportation. — Dunbar’s Chapters on Banking. — Financial Legislation. 3 hours.

Total 322: 1 Graduate, 50 Seniors, 114 Juniors, 116 Sophomores, 3 Freshmen, 38 Others.

Source: Harvard University. Report of the President of Harvard College, 1892-1893, p. 63.

 

 

1892-93.
ECONOMICS 1.
[Mid-Year Examination]

[Arrange your answers strictly in the order of the questions.]

  1. “Production, and productive, are of course elliptical expressions, involving the idea of something produced; but this something, in common apprehension, I conceive to be, not utility, but wealth.” Why should not all labor which produces utility, be accounted productive?
  2. “The distinction, then, between Capital and Non-Capital, does not lie in the kind of commodities, but in the mind of the capitalist.” Does pig-iron cease to be capital when the owner sells it and buys a country-house?
  3. What is meant when it is said that rent is no burden on the consumer?
  4. Why are the earnings of the professional classes higher than the wages of mechanics? Why are the wages of mechanics higher than those of day-laborers?
  5. Explain the connection between:

The tendency of profits to a minimum.
The law of diminishing returns.
The effective desire of accumulation.

  1. Is a general rise in prices advantageous to the community as a whole? to any part of it?
  2. Specie, bank-notes, inconvertible paper, checks, —are they or are they not “money”?
  3. “It is when the metals are completely superseded and driven from circulation that the difference between convertible and inconvertible paper begins to be operative.” Explain.
  4. Does foreign trade tend to bring about the same level of (1) money wages, (2) prices, in the trading countries?
  5. In what manner does a country gain when its exports increase?

Source: Harvard University Archives. Harvard University, Mid-year examinations, 1852-1943. Box 3, Bound Volume: Examination Papers, Mid-Year 1892-93.

 

1892-1893
ECONOMICS 1.
[Year-End Final Examination]

Division A.
[Arrange your answers strictly in the order of the questions.]

  1. What is meant when it is said that the rent paid for the use of a factory building enters into cost of production, while that paid for the use of the site does not?
  2. What determines the limits within which the foreign exchanges may fluctuate?
  3. According to Mill, “The universal elements of cost of production are the wages of labor and the profits of the capital.” Cairnes on the contrary says, “I repeat, therefore, that not only do wages not constitute the laborer’s share in the cost of production, but these can not be taken in any sense to represent that cost.”
    Why not?
  4. “It appears, therefore, that the fund available for those who live by labor tends, in the progress of society, while growing actually larger, to become a constantly smaller fraction of the entire national wealth.”
    Why?
  5. “The illusion which I am combatting, that Demand and Supply are independent economic forces, sometimes assumes another form in the notion that producers and consumers are distinct classes, and that production and consumption are acts which may go on irrespective of each other.”
    Explain the illusion.
  6. How is the price of wool in Australia likely to be affected by the shipment of frozen mutton to England?
  7. “In the language of Mr. Mill, ‘the produce of a country exchanges for the produce of other countries at such values as are required in order that the whole of her exports may exactly pay for the whole of her imports.’ Now, as a matter of fact, it very rarely happens that the whole exports of a country, even if we take an average of many years, exactly pay for the whole of its imports; nor can it be truly said that there is any tendency in the dealings of nations toward this result.” Why not?
  8. At what rate of interest did the United States borrow, when it exchanged 5-20 bonds for legal tender notes at par, in 1862-63?
  9. What do you infer from the success of distributive coöperation in Great Britain, as to the future development of coöperation in general?
  10. What are the grounds for saying that the general fall in prices in the United States in the period immediately after the civil war, had an effect on debtors different from that of the fall in prices since 1879?

 

Division B.
[Arrange your answers strictly in the order of the questions.]

  1. Does the benefit of international trade lie in the exports? in the imports?
    Why?
  2. Is a general fall in prices harmful to debtors? to creditors?
  3. Will an increase in the quantity of money in the community affect the rate of interest?
  4. How would you estimate the minimum reserve required by law to be anywhere held for deposits in country national banks of the United States?
  5. How would you explain the close correspondence in the banks of the United States between the amount of loans and the amount of deposits?
  6. Explain the decline in the volume of national bank notes in recent years.
  7. Explain why the original limit of uncovered issue for the Bank of England was put at £14,000,000.
  8. How would an act for the free coinage of silver in the United States at the present mint ratio, affect the price of silver bullion?
  9. Compare the attitude of the Latin Union toward the use of both metals in 1866 with its attitude in 1878.
  10. Compare carefully, as to the character and quantity of the issues of money provided for, the legislation of the United States in 1878 with that of 1890.
  11. Point out wherein profit-sharing is similar to coöperative production, wherein different.

Source: Harvard University Archives. Harvard University, Examination Papers, 1873-1915. Box 4, Bound Volume: Examination Papers, 1893-95, “Papers Set for Final Examinations in Philosophy, History, Government and Law, Economics, Fine Arts, and Music in Harvard College, June 1893”.

 

_________________________

1892-93.
ECONOMICS 2. ECONOMIC THEORY
 

Enrollment

[Economics] 2. Professor TAUSSIG. — Economic Theory. — Examination of selections from leading writers. 3 hours.

Total 38: 11 Graduates, 10 Seniors, 11 Juniors, 6 Others.

Source: Harvard University. Report of the President of Harvard College, 1892-1893, p. 63.

 

1892-93.
ECONOMICS 2.
[Mid-Year Examination]

  1. State George’s doctrine as to the cause of interest, and give an opinion of its soundness.
  2. “It may be said, we grant that wages are really paid out of the product of current industry, and that capital only affects wages as it first affects production, so that wages stand related to product in the first degree, and to capital in the second degree only; still, does not production bear a certain and necessary ratio to capital? and hence may not the measure of wages be derived from capital virtually, — though not, it is true, directly, — through its determination of product?” Consider whether so much would be granted by one holding to the wages-fund doctrine; and answer the questions.
  3. “The employer [in the West and South] advances to the laborer such provisions and cash as are absolutely required from time to time: but the ‘settlement’ does not take place until the close of the season or the year, and the final payment is often deferred until the crop is not only harvested but sold.” Under such conditions is it true that wages are paid out of capital, or limited in amount by the quantity of previously accumulated capital?
  4. What do you conceive President Walker’s opinion to be as to the effect on business profits of the possession of large means by the business man at the outset of his career?
  5. Are there grounds for saying that in a socialist community the conception of capital would be different from that in communities as now organized?
  6. Compare Adam Smith’s doctrine as to the relation of capital and wages with Ricardo’s.
  7. Compare Adam Smith’s conclusions with Ricardo’s as to the propriety of import duties levied to countervail internal taxes on necessaries consumed by laborers.
  8. “No extension of foreign trade will immediately increase the amount of value in a country, though it will very powerfully contribute to increase the mass of commodities, and therefore the sum of enjoyments.” What does Ricardo mean?
  9. “There is only one case, and that will be temporary, in which the accumulation of capital with a low price of food may be attended with a fall in profits.” What is the case, and why did Ricardo think it would be temporary?

Source: Harvard University Archives. Harvard University, Mid-year examinations, 1852-1943. Box 3, Bound Volume: Examination Papers, Mid-Year 1892-93.

 

1892-1893
ECONOMICS 2.
[Year-End Final Examination]

[One question in each of the three groups may be omitted.]

I.

  1. What is the meaning and importance of the proposition that demand for commodities is not demand for labor?
  2. How far is Ricardo’s doctrine as to the connection between labor and value similar to Marx’s doctrine that value consists of the labor incorporated in commodities?
  3. What is meant when it is said that the connection between value and expenses of production depends on the mobility of capital, while the connection between value and cost of production depends on the mobility of labor and capital?
  4. “The ideal of justice in distribution, applicable both to individual producers and to the different factors in production (land, labor, capital), may be stated thus: each should have a share in net income proportionate to the contribution which, by labor or by the use of material means of production, he has made to the product.”
    What should you say as to the feasibility of carrying out such a principle?

II.

  1. Explain briefly what is meant by total utility, marginal utility, and consumer’s rent.
    “Subject to these corrections, then, we may regard the aggregate of the money measures of the total utility of wealth as a fair measure of that part of the happiness which is dependent on wealth.” Mention one or two corrections.
  2. Give your opinion on the objection raised by Carey to the theory of rent, that the total rent paid for the use of land does not exceed interest at current rates on the total capital sunk in land.
  3. How far is it an answer to the proposition that rent and business profits are analogous, when it is said that the losses of some business managers must be set off against the larger gains of others?
  4. Explain Professor Marshall’s opinion as to the bearing on the relative wages of different laborers of
    1. The “rent” of labor;
    2. the standard of living among laborers;
    3. the expenses of production of labor;

and point out the connection between his views on these subjects.

III.

  1. Explain the distinctions (1) between private capital and social capital, (2) between historico-legal capital and national capital; and point out how far the two distinctions run on the same lines.
  2. “In the present condition of industry, most sales are made by men who are producers and merchants by profession. . . . For them, the subjective use-values of their own wares is, for the most part, very nearly nil. . . . In sales by them, the limiting effect which, according to our theoretical formula, would be exerted by the valuation of the last seller, practically does not come into play.”
    Explain what is meant, and consider the consequences as to the importance of the law that price is determined by the valuations of the marginal pairs.
  3. “Our whole interest is centred in the question as to the position which the law (of cost of production), so well accredited by experience, takes in the systematic theory of price. Does it run counter to our law of marginal pairs or not? Our answer is that it does not. It is as little of a contradiction as we before found to exist between the proposition that the marginal utility determines the height of subjective value, and the other proposition that the costs determine it.”
    In what way is the apparent contradiction removed in the two cases referred to by Böhm-Bawerk?
  4. Explain the three grounds on which Böhm-Bawerk bases the superiority of present over future goods, and give your opinion as to their relative importance and significance.

Source: Harvard University Archives. Harvard University, Examination Papers, 1873-1915. Box 4, Bound Volume: Examination Papers, 1893-95, “Papers Set for Final Examinations in Philosophy, History, Government and Law, Economics, Fine Arts, and Music in Harvard College, June 1893”.

_________________________

1892-93.
ECONOMICS 3. PRINCIPLES OF SOCIOLOGY

Enrollment

[Economics] 3. Mr. CUMMINGS. — The Principles of Sociology. — Development of the Modern State, and of its Social Functions. 3 hours.

Total 22: 5 Graduates, 9 Seniors, 4 Juniors, 1 Sophomore, 3 Others.

Source: Harvard University. Report of the President of Harvard College, 1892-1893, p. 63.

 

 

1892-1893
ECONOMICS 3.
[Mid-Year Examination]

Answer the questions in the order in which they stand. Omit two.

  1. “We have just seen that a one-sided application of the conception that society is of organic growth leads to difficulties, as well as the conception of artificial making. These we can only escape by recognizing a truth which includes them both.”
    What are these difficulties, and what is this truth?
  2. “If societies have evolved, and if that mutual dependence of parts which coöperation implies, has been gradually reached, then the implication is that however unlike their developed structures may become, there is a rudimentary structure with which they all set out.”
    What evidence do you find of such a structure?
  3. According to Aristotle, “Man is by nature a political” According to Thomas Aquinas, “homo est animal sociale et politicum.” How far is this insertion of “sociale” alongside of “politicum” significant of the different way in which the State presented itself to the mind of the Greek and to the mind of the medieval philosopher?
  4. “The theory of the social contract belongs in an especial manner to the political philosophers of the seventeenth and eighteenth centuries. But it did not originate with them. It had its roots in the popular consciousness of medieval society. As a philosophical theory, it had already been anticipated by the Greek Sophists.”
    Indicate briefly some of the important changes which the doctrine underwent.
  5. “In primitive societies the person does not exist, or exists only potentially, or, as we might say, in spe. The person is the product of the State.” Explain. What is the theoretical and historical justification of this doctrine, as against the contention that the individual loses what the State gains?
  6. Discuss the relative preponderance of free and of un-free elements at different stages of social development.
  7. It has been remarked by Spencer that those domestic relations which are ethically the highest, are also biologically and sociologically the highest. Discuss the historical evidence on this point. What is the test of this ethical superiority?
  8. To what extent is there ground for saying that the influence of militant and of industrial organization is traceable in the status of women and the duration of marriage in the United States and in other countries?

Source: Harvard University Archives. Harvard University, Mid-year examinations, 1852-1943. Box 3, Bound Volume: Examination Papers, Mid-Year 1892-93.

 

1892-1893
ECONOMICS 3.
[Year-End Final Examination]

[Answer the questions in the order in which they stand. Omit one.]

  1. “The different forms of the State are specifically divided, as Aristotle recognized, by the different conceptions of the distinction between government and subjects, especially by the quality (not the quantity) of the ruler.” Explain. Indicate briefly the relation of the different forms of the State to one another.
  2. “If there is any one principle which is clearly grasped in the present day, it is that political power is a public duty as well as a public right, that it belongs to the political existence and life of the whole nation, and that it can never be regarded as the property or personal right of an individual.” How far did this principle secure recognition in Greek, in Roman, and in medieval times?
  3. “The past seems to prove that kings and aristocracies make States, and that left to themselves, the people unmake them.” State carefully your reasons for agreeing or disagreeing with the political philosophy here involved.
  4. “This is one of curious phases of the railway problem in Europe, which has a tendency to show how multiform and various are the influences at work to modify and change the conditions of the railway problem, and how little can be gathered from mere government documents and laws to shed light upon this most interesting and intricate of all modern industrial questions.” What light does Italian, French and Austrian experience with railroads throw on the general question of State control?
  5. “Expediency and the results of experience must determine how far to go. They seem to justify public ownership of gas works, water works and electric lights. The same would doubtless be true of the telegraph and telephone.” Discuss the evidence.
  6. “We will first concentrate our attention on the economic kernel of socialism, setting aside for the moment the transitory aspect it bears in the hands of agitators, its provisional passwords, and the phenomena and tendencies in religion by which it is accompanied.” State and criticise this “economic kernel.”
  7. “The philanthropic and experimental forms of socialism, which played a conspicuous role before 1848, perished them[sic, “then”?] in the wreck of the Revolution, and have never risen to life again.” What were the characteristics of these earlier forms; and what was their relation to the movements which preceded them and followed them?
  8. How are the socialistic teachings of Lasalle and Marx related to the economic doctrines of Smith and Ricardo?
  9. What ground do you find for or against the contention that “socialism is the economic complement of democracy”?
  10. “Not only material security, but the perfection of human and social life is what we aim at in that organized co-öperation of many men’s lives and works which is called the State. . . . But where does protection leave off and interference begin?

Source: Harvard University Archives. Harvard University, Examination Papers, 1873-1915. Box 4, Bound Volume: Examination Papers, 1893-95, “Papers Set for Final Examinations in Philosophy, History, Government and Law, Economics, Fine Arts, and Music in Harvard College, June 1893”.

_________________________

1892-93.
ECONOMICS 4. ECONOMIC HISTORY OF EUROPE AND AMERICA

Enrollment

[Economics] 4. Mr. COLE. — Economic History of Europe and America since the Seven Years’ War. — Lectures and written work. 3 hours.

Total 116: 41 Seniors, 59 Juniors, 45 Sophomores, 3 Freshmen, 18 Others.

Source: Harvard University. Report of the President of Harvard College, 1892-1893, p. 63.

 

 

1892-93. ECONOMICS 4.
[Mid-Year Examination]

I.
[Take all.]

  1. State at least three parallels in the lives of Watt and George Stevenson.
  2. From an economic point of view, and assuming that a revolution must have come sooner or later, was the occurrence of the disturbances in France between 1785 and 1815 opportune or inopportune for France?
  3. Was there any necessary connection between the economic and the military reforms of Prussia between 1807 and 1812? If so, what?
  4. Why was the United States helped more than any other country by the introduction of steam navigation?

II.
[Omit one.]

  1. What was the origin and what were the main provisions of the English Corn Law of 1815?
  2. What were the main provisions of the French railway law of 1842?
  3. What were the main features of Gallatin’s plan for internal improvements in 1807?
  4. What was the social status at the beginning of this century of poor immigrants into America?
  5. What was the cause of the suspension of specie payments by the Bank of England in 1797?
  6. What was the effect of the Continental wars of 1793-1815 upon the English laborers? How was it manifested?
  7. What was the influence on French manufactures of Napoleon’s rise to power? Cite examples.

Source: Harvard University Archives. Harvard University, Mid-year examinations, 1852-1943. Box 3, Bound Volume: Examination Papers, Mid-Year 1892-93.

 

 

1892-93.
ECONOMICS 4.
[Year-End Final Examination]

[Arrange your answers strictly in the order of the questions.]

  1. What sort of wealth did France actually sacrifice in paying the German indemnity? What was the process?
  2. Explain the influence of the Civil War upon our tariff legislation.
  3. Explain whether or not England can obtain cheap coal from abroad after her own supplies become scarce.
  4. What motives had Congress for granting lands to Western railroads, — other than the Union Pacific? What was the system of grants adopted?
  5. It has been said that after 1850 England could not well maintain duties upon any class of imports, and hence free trade was inevitable. What do you think of the statement?
  6. Was the Zollverein an experiment in free trade or in protection? Why do you think as you do?
  7. How did the extraordinary demand for gold between 1871 and 1873 affect the rate of bank discount? How do you explain the effect?
  8. Show at least three important benefits arising from improved means of transportation.
  9. Explain carefully, but concisely, why the southern soils of the United States were rapidly exhausted before the war. Could resort have been made successfully to rotation of crops, to more careful cultivation, to the use of better tools?
  10. Using only the materials which have been furnished by Economics IV, show what in your opinion, after careful thought, England would have gained or lost up to the present time if the American colonies had not won their independence.

Source: Harvard University Archives. Harvard University, Examination Papers, 1873-1915. Box 4, Bound Volume: Examination Papers, 1893-95, “Papers Set for Final Examinations in Philosophy, History, Government and Law, Economics, Fine Arts, and Music in Harvard College, June 1893”.

_________________________

1892-93.
ECONOMICS 5. RAILWAY TRANSPORTATION

Enrollment

[Economics] 5. Professor TAUSSIG. — Railway Transportation. — Lectures and written work. 3 hours. 1st half-year.

Total 26: 7 Graduates, 12 Seniors, 5 Juniors, 1 Sophomores, 1 Other.

Source: Harvard University. Report of the President of Harvard College, 1892-1893, p. 63.

 

 

1892-93.
ECONOMICS 5.
[Mid-Year Final Examination]

[Arrange your answers strictly in the order of the questions].

  1. “A more powerful force than the authority of the courts was working against the Granger system of regulation. The laws of trade could not be violated with impunity. The effects were most sharply felt in Wisconsin. . . . In the second year of its operation [that of the law reducing rates], no Wisconsin road paid a dividend; only four paid interest on their bonds. Railroad construction came to a standstill. . . . Foreign capital refused to invest in Wisconsin; the development of the State was sharply checked; the very men who most favored the law found themselves heavy losers. . . . The very men who passed the law in 1874 hurriedly repealed it after two years trial.” State the essential features of the legislation here alluded to, and give an opinion as to this explanation of its effects.
  2. “The principle of tolls [rates based on cost of service] keeps rates up. If it is strictly applied, it makes it necessary that each item of business should pay its share of the fixed charges.” Why? or why not?
  3. What is meant when it is said that railway rates are governed by value of service?
  4. Compare :
    1. The natural system of rates.
    2. The German reform tariff.
    3. The maximum rates of the Granger legislatures.
  5. Is it true that the prohibition of pooling in the Interstate Commerce Act increases the severity of the long and short haul clause?
  6. What were the causes of the depression of 1888-90?
  7. Should you say that the approaching maturity of the Government debt gives a favorable opportunity for an experiment in public management, by the assumption of federal ownership of the Pacific roads?
  8. Give an opinion on two among the suggestions made by Mr. Clark as to future legislation on railways by the states.
  9. Sketch the history of railway policy in Italy.
  10. What have been the financial results of public railway management in Prussia?

Source: Harvard University Archives. Harvard University, Mid-year examinations, 1852-1943. Box 3, Bound Volume: Examination Papers, Mid-Year 1892-93.

_________________________

1892-93.
ECONOMICS 6. HISTORY OF U.S. TARIFF LEGISLATION

Enrollment

[Economics] 6. Professor TAUSSIG. — History of Tariff Legislation in the United States. 3 hours. 2d half-year.

Total 50: 7 Graduates, 19 Seniors, 17 Juniors, 3 Sophomores, 4 Others.

Source: Harvard University. Report of the President of Harvard College, 1892-1893, p. 63.

 

1892-1893
ECONOMICS 6.
[Year-End Final Examination]

[Answer all the questions, however briefly.]

  1. Sketch the industrial history of the country, and its bearing on tariff legislation, from 1816 to 1824.
  2. How far do protective duties account for the growth of the cotton manufacture from 1830 to 1840? Of the iron manufacture from 1840 to 1850? Of the silk manufacture from 1860 to 1880?
  3. What do you believe the state of public opinion to have been on tariff legislation in 1800? In 1824? In 1850?
  4. State the important provisions of the tariff act of 1857.
  5. Are there grounds for saying that the duty on pig iron since 1870 has proved a successful application of protection to young industries? State carefully what you think the test of success in such a case.
  6. What can be said for, what against, the change in the duties on sugar made in 1890?
  7. Compare the general character of the tariff act of 1883 with that of the act of 1890.
  8. Assume that, on the imposition of a duty on tin-plates, domestic production should so develop that tin-plates were made with less labor in the United States than in foreign countries. What would happen if thereafter the duty were removed?
  9. Explain what is the object, what the effect of minimum duties; and give two instances of their application, one before 1860, the other after.
  10. Explain briefly in what manner the connection between the tariff and wages was discussed by Webster in 1824, by Clay in 1824, and by Walker in 1845.

Source: Harvard University Archives. Harvard University, Examination Papers, 1873-1915. Box 4, Bound Volume: Examination Papers, 1893-95, “Papers Set for Final Examinations in Philosophy, History, Government and Law, Economics, Fine Arts, and Music in Harvard College, June 1893”.

_________________________

1892-93.
ECONOMICS 71. THEORY AND METHODS OF TAXATION

Enrollment

[Economics] 71. Professor DUNBAR. — The Theory and Methods of Taxation, with special references to local taxation in the United States. 3 hours. 1st half-year.

Total 21: 7 Graduates, 9 Seniors, 2 Juniors, 1 Sophomore, 2 Others.

Source: Harvard University. Report of the President of Harvard College, 1892-1893, p. 63.

 

 

1892-93.
ECONOMICS 7[1].
[Mid-Year Final Examination]

[Let your answers stand in the order of the questions.]

  1. What is Mr. Bastable’s theory (pp. 339-342) as to the incidence of a tax on a commodity, and what are the conditions on which he finds that any shifting of the tax will depend? How far does his doctrine in this particular differ from that of Ricardo?
  2. Discuss the following extract from Leroy-Beaulieu (Science des Finances, II. 303):—
    So the land tax, unless it is extraordinarily high or very badly assessed, has no influence on the price of agricultural products: it merely diminishes what in scientific language is called the rent of land, — that is the net income of the landowner after deducting the expenses of cultivation and the profits of the farmer. This proposition is generally true in all countries where the land is completely occupied: it does not apply, on the contrary, to new countries where a large part of the soil is not yet under cultivation, like the United States or Australia. In these countries the land tax acts as an increase of the general cost of working new lands, and consequently retards their reduction to cultivation.
  3. In answer to the demand for taxes resting exclusively or chiefly on land and its great unearned increment of value, it is sometimes urged that gains are often offset by losses, and that individuals can hardly be called on to give up their surplus gains unless they are guaranteed against possible loss. How much weight is to be attached to this answer?
  4. Discuss Bastable’s remark that,—
    It may be urged that progressive taxation is not in fact likely to weaken the disposition to save. It will only affect those who possess a good deal already, and such persons save as much from habit as from conscious motive. There is, too, the further fact that the heavier taxation on the rich will leave the poor a larger disposable sum, part of which they may save, and to that extent increase the store of wealth.
  5. What do you say to the proposition maintained by Mill (Book V., ch. ii. §4) and discussed by Bastable (p. 297), that the part of income which is saved should be exempt from taxation?
  6. W. is credited with having laid down two propositions: First, that “any income tax which permits of any exemption whatever is a graduated income tax”; and, secondly, that “a graduated income tax to the extent of its discrimination is an act of confiscation.”
  7. State the general plan on which the French Contribution des Patentes is levied, and then discuss the following:—
    1. Bastable says (p. 411):—
      The Patente is very far from being a proportional tax on industrial gains. It rather resembles a charge on certain necessaries of the business, such as buildings, labor, or motive power.
    2. Leroy-Beaulieu (Science des Finances, I. p. 396) says:—
      The manufacturer being taxed by the general tax on rents [personelle-mobilière] there is evident injustice in loading him with an additional tax on his habitation.
    3. And in general he says (ibid., p. 380):—
      In countries like France where incomes in general are not subject to any special direct tax, it is indisputable that a tax on the profit of manufacturers, of merchants, and of the liberal professions, has no reason for existence and can only be explained by the brutal law of fiscal necessity.
  8. Describe the changes which the Prussian income tax has gone through and the distinctive characteristics of the law of 1891.
  9. Describe the plan on which the English “death duties” are now arranged.
  10. What are the methods used in different countries for the taxation of tobacco, and how far does each appear applicable in the United States?
  11. Which of the following taxes are best fitted for national use and which for local, and why?

Excise;
Income;
Real Estate;
Stamps on deeds, commercial paper and legal instruments;
Successions.

Source: Harvard University Archives. Harvard University, Mid-year examinations, 1852-1943. Box 3, Bound Volume: Examination Papers, Mid-Year 1892-93.

_________________________

1892-93.
ECONOMICS 72. FINANCIAL ADMINISTRATION AND PUBLIC DEBTS

Enrollment

[Economics] 72. Professor DUNBAR. — Financial Administration and Public Debts. 3 hours. 2d half-year.

Total 23: 10 Graduates, 8 Seniors, 3 Juniors, 1 Sophomores, 1 Other.

Source: Harvard University. Report of the President of Harvard College, 1892-1893, p. 63.

 

1892-1893
ECONOMICS 7[2].
[Year-End Final Examination]

[Give one half of your time to a careful treatment of the questions under A.]

A.

  1. Discuss the conditions necessary for maintaining a thoroughgoing budget system, and show what changes (if any) of constitution, law, or political practice would be required, in order to set such a system in operation in the United States.
  2. When the United States issued the 5-20 bonds (principal and interest payable in gold), they had the choice between three courses, viz:
    1. To sell the bonds for par in gold and make the rate of interest high enough to attract buyers;
    2. to sell the bonds for gold at such discount as might be necessary, their interest being fixed at six per cent;
    3. to sell the bonds for their nominal par in depreciated paper.
      Which of these courses now seems to you the best, and why?
  3. Discuss the following:
    “Viewed as a purely financial question, it is no occasion for congratulation that a debt is widely diffused. Not only is its management necessarily more expensive, but the facility offered to politicians to use the debt for party and personal ends often defeats the best purposes of the financier. . . .France, for example, continued to pay for a number of years a higher rate of interest than was necessary, because the government feared the voting power of the holders of rentes. Nor do industrial considerations necessarily lead to the approval of widely-diffused debt. The unfailing indication of healthy state of industries is found in the personal attention of all members of society to business affairs. and this can only come with personal interest in some particular form of product. In so far as the private income of individuals arises from payments of interest by the state, the public is deprived of the beneficial workings of that solicitous care which insures success in industrial ventures. —Adams, Public Debts, p. 43.

B.

  1. State the manner in which the selling value of bonds is influenced, by, —
    1. annual drawings by lot for payment;
    2. reserved right to pay at pleasure;
    3. agreement to pay at or after some distant date;
    4. arrangement like that of the “Five-twenties.”
  2. The distinction between a bond reimbursable from the date of issue and one which is secured against redemption for ten or twenty years, is said to be “one of the most fundamental that presents itself in the entire course of credit operations,” because,—
    “A bond reimbursable from the date of its issue shows great carelessness on the part of the administration as to ultimate payment; on the other hand, a bond guaranteed against immediate payment is evidence of an intention to escape the evils of perpetual indebtedness.” — Adams, Public Debts, p. 161.
    Give your reasons for agreeing or for disagreeing with this statement.
  3. Under what conditions is the use of terminable annuities as a species of sinking fund advisable, and on what principle should the extent to which their use is carried be limited, if at all?
  4. What do you say to the following dictum as to buying public debt:
    “Payment by purchase (of bonds) upon the market at market prices is defensible when bonds are below par, but not when above par and so conditioned as to be payable, within a reasonable time, at their nominal value.”
  5. Describe the operation by which the French government converted the Morgan Loan in 1875 and state any criticism to be made upon this conversion.

Source: Harvard University Archives. Harvard University, Examination Papers, 1873-1915. Box 4, Bound Volume: Examination Papers, 1893-95, “Papers Set for Final Examinations in Philosophy, History, Government and Law, Economics, Fine Arts, and Music in Harvard College, June 1893”.

_________________________

1892-93.
ECONOMICS 8. HISTORY OF U.S. FINANCIAL LEGISLATION

Enrollment

[Economics] 8. Professor DUNBAR. — History of Financial Legislation in the United States. 2 hours. 1st half-year.

Total 34: 3 Graduates, 19 Seniors, 11 Juniors, 1 Sophomore.

Source: Harvard University. Report of the President of Harvard College, 1892-1893, p. 63.

 

1892-93.
ECONOMICS 8.
[Mid-Year Final Examination]

[Let your answers stand in numerical order.]

  1. Hamilton has sometimes been charged with favoring the policy of a perpetual public debt. Discuss the grounds for this charge.
  2. The act of 1790 for assuming the debts of the States did not wait for the settlement of accounts between the States and the Union. Did this failure to wait necessarily affect the result?
  3. What is a “direct tax” of the United States and on whom and how is it laid? Give instances.
  4. What were the provisions of law or the practices in use, regulating the kinds of currency received and paid by the Treasury, between 1789 and 1846?
  5. What was the Specie Circular, and what were its effects?
  6. What is Mr. Gallatin’s view of the part played by the United States Bank and of its influence, in the disastrous period 1837-1841?
  7. Describe the Independent Treasury Act of 1846, and state any modifications that the system has undergone.
  8. Give some account of the treasury notes issued 1837-46, and show how they resembled or differed from notes now issued by the United States.
  9. What considerations are there which tend to create doubt as to the necessity of the first Legal Tender Act?
  10. State the decisions of the Supreme Court of the United States on the constitutionality of the Legal Tender Acts.
  11. Sketch the legislation which has established the national banking system.
  12. Describe the change which has taken place in the meaning attached to the word “resumption,” and the circumstances which have given us a legal tender currency of fixed amount.

Source: Harvard University Archives. Harvard University, Mid-year examinations, 1852-1943. Box 3, Bound Volume: Examination Papers, Mid-Year 1892-93.

_________________________

1892-93.
ECONOMICS 9. SOCIAL AND ECONOMIC CONDITION OF WORKINGMEN

An earlier post to this course with valuable links to the works quoted in the exams.

Enrollment

[Economics] 9. Mr. CUMMINGS. — The Social and Economic Condition of Workingmen in the United States and in other countries. 3 hours.

Total 24: 3 Graduates, 10 Seniors, 7 Juniors, 4 Others.

Source: Harvard University. Report of the President of Harvard College, 1892-1893, p. 68.

 

1892-93.
ECONOMICS 9.
[Mid-Year Examination]

[Arrange your answers in the order in which the questions stand. So far as possible illustrate your discussions by a comparison of the experience of different countries. Omit two questions.]

  1. “In a society adjusted to manual labor, it is absolutely impossible that a labor problem, as a class problem, should take its origin; but in a society adjusted to machinery, provided the English law of property be maintained, the development of class lines will surely make its appearance in industries.”
    State fully your reasons for agreeing or disagreeing with these assertions.
  2. “First, government must regulate the plane of competition, for without legal regulation the struggle between men for commercial supremacy will surely force society to the level of the most immoral man who can maintain himself.”
    What evidence does the history of factory legislation furnish upon these points?
  3. Comment upon the following passage: “The object held in view by workmen, when they organized themselves into unions, was to gain again that control over the conditions of labor which they lost when machinery took the place of tools.”
  4. “The English public has had the courage and strength to leave workingmen’s associations full freedom of movement, at the risk even of temporary excesses and acts of violence, such as at one time stained the annals of trades-unions.” Explain.
    How far is this true of France? Of the United States?
  5. Describe briefly the origin, growth, and present tendencies of the English Friendly Society movement.
  6. To what extent do trade organizations and friendly societies constitute an aristocracy of labor?
  7. To what forms of remuneration can the evils of “sweating” be traced?
  8. “The aim of Coöperation is at the same time the aim of Trade Unionism.” In what sense?
  9. Sketch briefly the course of factory legislation during the present century either in England or in the United States.
  10. Comment on the following passage: “The fact that the ignorant masses are enabled by the factory to engage in what it once took skilled labor to perform has given the widespread impression that factory labor has degraded the skilled, when in truth it has lifted the unskilled; and this is the inevitable result of the factory everywhere.”

Source: Harvard University Archives. Harvard University, Mid-year examinations, 1852-1943. Box 3, Bound Volume: Examination Papers, Mid-Year 1892-93.

 

1892-93.
ECONOMICS 9.
[Year-End Final Examination]

[Arrange your answers in the order in which the questions stand. So far as possible illustrate your discussions by a comparison of the experience different countries. Omit two questions.]

  1. How is the burden of contribution distributed in each of the three departments of the German system of compulsory insurance? What theoretical or practical objections have you to the system?
  2. “In England especially the State is not in a position to compete effectively with energetic Insurance Companies or with the Friendly Societies, pulsating with the vigour of social life; and still less can it so compete when hampered by restrictions which handicap its powers.” Discuss the evidence on this point furnished by English experience with government workingmen’s insurance. Are there any indications that German ideas are gaining ground in England?
  3. “What, we will ask, is the relation of Profit-sharing to the ordinary wage system; and to what extent does Profit-sharing constitute an improvement upon the ordinary wage system?” Are there grounds for the assertion that Profit-sharing is “inferior in point of equity and expediency to the ordinary non-coöperative wage system“?
  4. “Besides the militant trade unionist workmen, that very shrewd class of workingmen, the coöperators, regard Profit-sharing with marked disapprobation; so much so that, although Profit-sharing forms an essential part of the professed principles of Industrial Coöperation, yet by far the greater part of Industrial Coöperation is carried on upon the system of altogether excluding the employees from participation in profits.” What are the facts referred to, and how do you account for them?
  5. “Here it is necessary to interpolate a protest against the assertion almost universally made by previous writers on this subject, that ‘Industrial Coöperation has succeeded in distribution, but has failed in production,’ — an assertion generally coupled with the explanation that ‘production’ is too difficult to be, as yet, undertaken by workingmen.” What are the facts?
  6. “But the enthusiastic Coöperator will ask: why not develop the voluntary system of democratic Coöperation until it embraces the whole field of industry?” What do you conceive to be the economic limits to such extension by consumers’ associations?
  7. “Having considered the social and economic position of workers in the coal, iron and steel industries in several countries, let us now by proper combination ascertain the average conditions prevailing in the two continents.” What are the probable conclusions to be drawn from these comparative statistics of family budgets in the United States and other countries?
  8. “The Hungarians, Italians, Bohemians and Poles, who throng our gates give most concern. . . . Up to the present time there seems no ground to fear that such new comers have wielded a depressing influence. There seems rather reason for congratulation in the fact that instead of their having lowered the American standard of living, the American standard of life has been raising them.” Discuss the evidence. What light do recent changes in the character and volume of migration from different countries throw on this problem?
  9. Indicate briefly the course of short-hour legislation in Massachusetts. How does it compare with the legislation in other states and other countries?
  10. Indicate carefully how far there has been any approximation to compulsory arbitration in Massachusetts; in New York; in other countries. What are the objections to compulsory arbitration?
  11. What do you conceive to be the significance of the Farmers’ Alliance and the Single Tax movements in the United States? And how are they related to each other?
  12. Precisely what evidence is there for and against the contention that the employment of “private armed forces” has been largely responsible for violence and bloodshed during strikes? Give concrete examples. 

Source: Harvard University Archives. Harvard University, Examination Papers, 1873-1915. Box 4, Bound Volume: Examination Papers, 1893-95, “Papers Set for Final Examinations in Philosophy, History, Government and Law, Economics, Fine Arts, and Music in Harvard College, June 1893”.

_________________________

1892-93.
ECONOMICS 10. U.S. AND EUROPEAN ECONOMIC HISTORY TO 1763

Enrollment

[Economics] 10. Professor ASHLEY. — The Economic History of Europe and America, to 1763. 3 hours.

Total 20: 6 Graduates, 7 Seniors, 4 Juniors, 3 Sophomores.

Source: Harvard University. Report of the President of Harvard College, 1892-1893, p. 63.

 

1892-93.
ECONOMICS 10.
[Mid-Year Examination]

N.B. — Not more than seven questions must be attempted.

  1. Present the substance of recent suggestions as to the origin of the Celtic Sept, and compare them with earlier views.
  2. Describe the Roman villa system, and compare it with mediaeval manorial agriculture and with modern American farming.
  3. Discuss the value of the Domesday Survey for economic history.
  4. Explain the importance of “Commutation.”
  5. State and criticize Mr. Thorold Rogers’ view of the causes of the Peasant Revolt of 1381.
  6. Describe the character of internal trade in England in the twelfth and thirteenth centuries.
  7. To what extent was the mediaeval regulation of industry justified?
  8. Trace the history, and comment on the significance, of Journeymen’s Societies.
  9. “Capital is a historical category.” statement in the light of medieval history. Explain and criticize this statement in the light of mediaeval history.

Source: Harvard University Archives. Harvard University, Mid-year examinations, 1852-1943. Box 3, Bound Volume: Examination Papers, Mid-Year 1892-93.

 

1892-93.
ECONOMICS 10.
[Year-End Final Examination]

[Candidates are requested to attempt only six questions, — of which six one at least must be chosen from the first set.]

  1. Compare the position of the Roman coloni with that of the peasants of the Middle Ages.
  2. “The reign of Edward I appears to mark the turning-point in the history of the craft-gilds.” Explain and criticize this.
  3. Estimate the importance of the work of M. Fustel de Coulanges in relation to Economic History.
    ________________________________
  4. How did the Reformation affect the English craft companies?
  5. Describe the “domestic system” of industry, and compare it with earlier and later systems.
  6. Narrate the later fortunes of the Hanseatic merchants in England.
  7. State and discuss the principles involved in the Poor Law of the sixteenth century.
  8. Give some account of the various discussions concerning economic policy occasioned by the East India Company.
  9. Compare a New England town with an English manor.

Source: Harvard University Archives. Harvard University, Examination Papers, 1873-1915. Box 4, Bound Volume: Examination Papers, 1893-95, “Papers Set for Final Examinations in Philosophy, History, Government and Law, Economics, Fine Arts, and Music in Harvard College, June 1893”.

_________________________

1892-93.
ECONOMICS 11. HISTORY OF ECONOMIC THEORY BEFORE ADAM SMITH
 

Enrollment

[Economics] 11. Professor ASHLEY. — History of Economic Theory, down to Adam Smith. 2 hours. 2d half-year.

Total 8: 7 Graduates, 1 Senior.

Source: Harvard University. Report of the President of Harvard College, 1892-1893, p. 63.

1892-93.
ECONOMICS 11.
[Year-End Final Examination]

[Candidates are requested to attempt only six questions.]

  1. Explain Aristotle’s view of chrematistics.
  2. What has been the economic influence of the Roman law?
  3. Compare the fundamental ideas of the Canonists with those of the Socialists.
  4. Explain the Canonist doctrine of Partnership.
  5. What were the principles involved in the discussion concerning Montes Pietatis?
  6. Consider the influence of the Reformation on Economic opinion.
  7. Sketch briefly the various stages in the history of Mercantilism.
  8. Explain briefly the significance for the history of economic thought of either Bodin, or Sir Josiah Child.

Source: Harvard University Archives. Harvard University, Examination Papers, 1873-1915. Box 4, Bound Volume: Examination Papers, 1893-95, “Papers Set for Final Examinations in Philosophy, History, Government and Law, Economics, Fine Arts, and Music in Harvard College, June 1893”.

 

Categories
Exam Questions Harvard

Harvard. Final examinations in Political Economy courses, 1891-1892

 

HARVARD. ECONOMICS EXAMINATIONS, 1891-1892

With the start of the 2021-22 academic year Economics in the Rear-view Mirror resumes the careful transcription of documents for the digital record of the development of economics education.

The Harvard archives are full of exam materials across time and fields so I pick up with where I left off in that series. Edward Cummings joined the teaching staff that in 1891-92 only consisted of two professors (Dunbar and Taussig) and a pair of instructors (Edward Cummings and William M. Cole).

_______________________

Note to self: Still Missing for 1891-92.

Political Economy 3. Edward Cummings. Mid-year examination, 1892
Political Economy 4. William M. Cole. Mid-year examination, 1892
Political Economy 7. Charles F. Dunbar. Mid-year examination, 1892

_______________________

1891-92
POLITICAL ECONOMY 1.
Course Description and Enrollment.

Primarily for Undergraduates:—

[Political Economy] 1. Professor [Frank W.] Taussig, Mr. [William M.] Cole, and Mr. [Edward] Cummings.

— First half-year: Mill’s Principles of Political Economy. 3 hours.

— Second half-year:

Division A (Theoretical): Mill’s Principles of Political Economy. — Cairnes’s Leading Principles of Political Economy. 3 hours.

Division B (Descriptive): Lectures on Finance, Labor and Capital, Coöperation. — Hadley’s Railroad Transportation.—Dunbar’s Chapters on Banking. 3 hours.

Total 288: 1 Graduate, 47 Seniors, 102 Juniors, 91 Sophomores, 7 Freshmen, 40 Others.

Source:   Harvard University, Annual Reports of the President and Treasurer of Harvard College, 1891-92, p. 54.

 

1891-92
POLITICAL ECONOMY 1.
Mid-Year Examination, 1892.

[Arrange your answers strictly in the order of the questions. Divide your time equally between the two parts of the paper.]

I.
[Omit one.]

  1. Mill says that “the laws and conditions of the production of wealth partake of the character of physical truths. . . . Whatever mankind produces must be produced in the modes, and under the conditions, imposed by the constitution of external things, and by the inherent properties of their own bodily and mental structure.” Is this true of the laws and conditions of production from land? of the laws and conditions of the accumulation of capital?
  2. Of things limited in quantity, it is said that “their value depends on the demand and the supply. . . . But the quantity demanded is not a fixed quantity, even at the same time and place; it varies according to the value; if the thing is cheap, there is usually a demand for more of it than when it is dear. The demand therefore partly depends on the supply. But it was before laid down that the value depends on the demand. From this contradiction, how shall we extricate ourselves? How solve the paradox, of two things, each depending on the other?”
  3. “Every fall in profits lowers in some degree the value of things made with much or durable machinery, and raises that of things made by hand; and every rise in profits does the reverse.” Explain.
  4. Is there any inconsistency between the propositions that the value of money depends,
    (1) on its cost of production at the mines;
    (2) on its quantity;
    (3) on the expansion and contraction of credit;
    (4) on the terms on which a country gets its imported commodities.
  5. Explain Mill’s reasoning (1) as to the manner in which an issue of inconvertible paper money drives specie out of circulation; (2) as to the manner in which, under a double standard, one metal [which one?] disappears from circulation. Are the results, in fact, brought about in the manner described by Mill?
  6. Explain carefully how a decrease in the foreign demand for a country’s exports causes loss to those who consume its imports.

II.
[Answer all, briefly.]

  1. Does nature give more aid to man in one kind of industry than in another?
  2. Are there grounds for saying that the necessity of restraining population is confined to a state of inequality of property?
  3. What are the advantages and disadvantages of a currency composed of specie, as compared with one of equal amount composed of inconvertible paper money?
  4. What are the laws of value applicable to (1) silver bullion; (2) iron nails; (3) wool; (4) eighteenth century furniture?
  5. Does the benefit of foreign trade consist in its affording an outlet for the surplus produce of a country?
  6. Mill says the superiority of reward in certain occupations may be the consequence of competition, and may be due to the absence of competition. Explain which explanation holds good of the high wages (1) of laborers in whom much confidence is reposed; (2) of laborers in disagreeable employments; (3) of laborers whose education has been expensive.
  7. What is the nature of the remuneration received by (1) a manufacturer on a large scale; (2) an independent artisan; (3) a farmer tilling land which he has leased at a fixed rent; (4) the owner, of a building who receives rent from those using the building.

Source: J. L. Laughlin, Economics 1: A Synopsis of John Stuart Mill’s Principles of Political Economy (Cambridge, MA: W.H. Wheeler, 1892), pp. 101-103.

 

1891-92
POLITICAL ECONOMY 1.
Division A.

Final Examination, 1892.

[Arrange your answers strictly in the order of the questions.]

I.
[Omit one.]

  1. “It will be remembered that in a former portion of this work I criticized at some length the received doctrine of Cost of Production, which, as expounded by Mr. Mill and others, is represented as consisting in, and varying with, the wages and profits of producers. I stated then that this conception of cost was not reconcilable with the doctrine of international values upheld by the same authorities, which refers these phenomena, not to cost of production, but to the reciprocal demand of exchanging nations.” Why not reconcilable?
    [John Elliott Cairnes, Some Leading Principles of Political Economy Newly Expounded (New York: Harper & Brothers, 1874), p. 343. https://hdl.handle.net/2027/loc.ark:/13960/t85h88k6k?urlappend=%3Bseq=351]
  2. “The actual price, therefore, of any given commodity will, it is evident, be the composite result of the combined action of these several agencies”—namely, reciprocal international demand, reciprocal domestic demand, and cost of production. Explain.
    [John Elliott Cairnes, Some Leading Principles of Political Economy Newly Expounded (New York: Harper & Brothers, 1874), p. 94. https://hdl.handle.net/2027/loc.ark:/13960/t85h88k6k?urlappend=%3Bseq=102]
  3. “Assuming a certain field for investment, and the prospect of profit in this such as to attract a certain aggregate of capital, and assuming the national industries to be of a certain kind, the proportion of this aggregate capital which shall be invested in wages is not a matter within the discretion of capitalists, always supposing they desire to obtain the largest practical return upon their outlay.” Why?
    [John Elliott Cairnes, Some Leading Principles of Political Economy Newly Expounded (New York: Harper & Brothers, 1874), p. 186. https://hdl.handle.net/2027/loc.ark:/13960/t85h88k6k?urlappend=%3Bseq=194]
  4. “We see, then, within what very narrow limits the possibilities of the laborer’s lot are confined, so long as he depends for his well-being upon the produce of his day’s work. Against these barriers Trades-unions must dash themselves in vain.” What, according to Cairnes, are the barriers?
    [John Elliott Cairnes, Some Leading Principles of Political Economy Newly Expounded (New York: Harper & Brothers, 1874), p. 283. https://hdl.handle.net/2027/loc.ark:/13960/t85h88k6k?urlappend=%3Bseq=291]
  5. “Saving (for productive investment), and spending, coincide very closely in the first stage of their operations.” Explain Mill’s meaning.

II.
[Answer all.]

  1. Why is there a tendency of profits to a minimum?
  2. What is the effect of a rise in the value of money on debtors and on creditors?
  3. “Though laborers in certain departments of industry are practically cut off from competition with laborers in other departments, the competition of capitalists, as I have already pointed out, is effective over the whole field.” How is this consistent with the existence of large amounts of Fixed Capital?
  4. “And here this remark may at once be made: that as the course of price in the field of raw products is, on the whole, upward, so in that of manufactured goods the course is, not less strikingly, in the opposite direction. The reasons of this are exceedingly plain.” (Cairnes.) What are they?
  5. What would be the effect on wages and profits of the universal adoption of coöperative production? of profit-sharing ?
  6. How far did the premium on gold during the civil war measure the real depreciation of the paper?
  7. Compare Mill’s attitude on coöperation with Cairnes’s.

 

Source: Harvard University Archives. Harvard University Examination Papers, 1873-1915. Box 3, Papers Set for Final Examinations in Philosophy, Political Economy, History, Roman Law, Fine Arts, and Music in Harvard College (June, 1892) in the bound volume: Examination Papers 1890-92.

Also J. L. Laughlin, Economics 1: A Synopsis of John Stuart Mill’s Principles of Political Economy (Cambridge, MA: W.H. Wheeler, 1892), pp. 106-108.

 

1891-92
POLITICAL ECONOMY 1.
Division B.

Final Examination, 1892.

[Arrange your answers strictly in the order of the questions]

  1. What is the fundamental objection to the issue of inconvertible paper money? What light is thrown on it by the experience of the United States during the civil war?
  2. Is a rise in the value of money advantageous to debtors or to creditors, or to neither? Why?
  3. What is the cause of the tendency of the rate of interest to fall?
  4. What would be the effect upon the price of food, and upon rent, of a tax of a fixed sum per acre upon agricultural land?
  5. Taking the two following accounts as representing the condition of a bank at different dates, state (1) what operations are most likely to have given rise to the changed condition, and (2) whether the bank is American or foreign, city or country, with your reasons for thinking so:—
I.
Capital 100,000 Government securities 5,000
Surplus 10,000 Other securities 50,000
Profits 3,000 Loans 255,000
Notes 20,000 Expenses 2,000
Deposits 250,000 Cash 71,000
383,000 383,000

 

II.

Capital 100,000 Government securities 5,000
Surplus 12,000 Other securities 50,000
Profits 2,000 Loans 260,000
Notes 20,000 Expenses 1,000
Deposits 270,000 Cash 88,000
404,000 404,000

 

  1. What is the sliding scale of discount? Name two countries in which it is used.
  2. Point out wherein there are differences, wherein similarities, in the legal provisions of the United States, England, and France, for the security, immediate and ultimate, of bank notes.
  3. Give a brief history of the small change (under one dollar) in the United States since 1850.
  4. What would be the effect upon the price of silver bullion of an act for free coinage of silver?
  5. What was the nature and purpose of the original restriction upon the amount of national bank notes in the United States? When and why was it repealed?
  6. Compare the main features of the silver acts of 1878 and 1890.
  7. How are profits divided in schemes for distributive coöperation? For credit coöperation? What is the important difference?

Source: Harvard University Archives. Harvard University Examination Papers, 1873-1915. Box 3, Papers Set for Final Examinations in Philosophy, Political Economy, History, Roman Law, Fine Arts, and Music in Harvard College (June, 1892) in the bound volume: Examination Papers 1890-92.

_______________________

1891-92
POLITICAL ECONOMY 2.
Course Description and Enrollment

For Graduates and Undergraduates:—

[Political Economy] 2. Professor [Frank W.] Taussig. — Economic Theory. — Examination of selections from leading writers. 3 hours.

Total 38: 9 Graduates, 17 Seniors, 8 Juniors, 1 Sophomore, 1 Freshman, 2 Others.

Source:   Harvard University, Annual Reports of the President and Treasurer of Harvard College, 1891-92, p. 54.

 

1891-92
POLITICAL ECONOMY 2.
Mid-Year Examination, 1892.

[Arrange your answers strictly in the order of the questions]

  1. Are laborers paid out of the product of their own labor in cases where the employer sells the product before pay-day and pays the laborers out of the proceeds?
  2. “Whether wages are advanced out of capital in whole, or in part, or not at all, it still remains true that it is the product to which the employer looks to ascertain the amount which he can afford to pay: the value of the product furnishes the measure of wages. . . .
    It is the prospect of a profit in production which determines the employer to hire laborers; it is the anticipated value of the product which determines how much he can pay them.”
    Is this consistent with the wages-fund theory?
  3. Consider the following: —

“Given machinery, raw materials, and a year’s subsistence, does it make no difference with the annual product whether the laborers are Englishmen or East-Indians? Certainly if one quarter part of what has been adduced under the head of the efficiency of labor be valid, the difference in the product of industry arising out of differences in the industrial quality of distinct communities of laborers are so great as to prohibit us from making use of capital to determine the amount that can be expended in any year or series of years in the purchase of labor.”

  1. How does President Walker prove the existence of a no-profits class of business men?
  2. Wherein does President Walker’s theory of distribution differ from Professor Sidgwick’s?
  3. What grounds are there for saying that Political Economy is distinctly a modern science?
  4. “Let us suppose, for example, that in the greater part of employments the productive powers of labour had been improved to ten-fold, or that a day’s labour could produce ten times the quantity of work which it had done originally; but that in a particular employment they had been improved only to double, or that a day’s labour could produce only twice the quantity of work it had done before. In exchanging the produce of a day’s labour in the greater part of employments for that of a day’s labour in this particular one, ten times the original quantity of work in them would purchase only twice the original quantity in it. Any particular quantity in it, therefore, a pound weight, for example, would appear to be five times dearer than before. In reality, however, it would be twice as cheap.” — Wealth of Nations, Book I. ch. viii.
    Explain what Adam Smith meant; and what Ricardo would have said as to this passage.
  5. Explain Adam Smith’s conclusions as to the effect on wages, profits, and rent, of the progress of society; noting briefly the reasoning which lead to the conclusion in each case.
  6. Examine the following criticisms on Malthus: —
    1. that there is no such difference of law between the increase of man and of the organic beings which form his food, as is implied in the proposition that man increases in a geometrical, food in arithmetical ratio;
    2. that the adaptation of numbers to the means available for their support is effected by the felt or anticipated pressure of circumstances and the fear of social degradation, within a tolerable degree of approximation to what is desirable.
  7. Explain carefully Ricardo’s doctrine as to the effect of profits on value.

Source: Harvard University Archives. Frank Taussig’s Scrapbook of his examinations. Posted earlier in Economics in the Rear-view Mirror.

1891-92
POLITICAL ECONOMY 2.
Final Examination, 1892.

[Arrange your answers strictly in the order of the questions]

  1. Does the example of a laborer hired by a farmer, and paid by him at the close of the season, after the crop has been harvested and disposed of, present a case of labor paid, not out of capital, but out of the product of current industry?
  2. What do you conceive the relation of political economy to laissez faire to have been with Adam Smith? with Ricardo and his contemporaries? How would you state the relation yourself?
  3. “Ricardo never fairly appreciated that his notion of the laborer’s ‘necessaries’ stood for something subject to wide variation in different stages of civilization. It is true that in one passage he says with emphasis that the necessaries, which determine the natural rate of wages, depend on habits which vary with time and place; but elsewhere he sets up a distinction between gross and net income, which is tenable only if we put the laborer’s necessaries side by side with other elements of cost of production. The distinction loses its practical harshness, when he admits that the laborer may at times receive, over and above natural wages, some part of the community’s net income; but its theoretic shortcomings then become the more obvious.” (Cohn, National-oekonomie.)
    Explain Ricardo’s conception of natural wages and net income, here referred to; and examine the justice of this criticism.
  4. “The average rate of profits is the real barometer, the true and infallible criterion of national prosperity. A high rate of profit is the effect of industry having become more productive, and it shows that the power of society to amass capital, and to add to its wealth and population, has been increased.” (M’Culloch’s Political Economy.) What led to the adoption of this test by M’Culloch? Should you accept it?
  5. What were Ricardo’s views as to the effect of foreign trade on profits?
  6. “In the actual period of production, on a wages system, the existing supplies for laborers are distributed to laborers in wages, while they, with the help of fixed capital, till the ground and work up the raw materials, transforming the old capital into a new product. . . . The product is divided at the end of the period of production into the replacement of capital (support of laborers, raw material, and wear of fixed capital), profits, and rent. . . . Hence it is clear that wages and profits are not parts of the same whole. Wages were in capital at the beginning of the period of production; profits are in product at its close.” (W. G. Sumner.)“We may suppose that share of the National Dividend which goes as rent to be set on one side; and then there remains what would be produced by labour and capital if they were all applied under conditions no more favourable than those under which they were applied at the margin of profitable employment; and a proposal was made by the present writer, in the Economics of Industry, that this should be called the Wages-and-Profits Fund, or the Earnings-and-Interest Fund. These terms were suggested in order to emphasize the opinion that the so-called Wages-Fund theory, however it might be purified from the vulgar errors which had grown around it, still erred in suggesting that earnings and interest, or wages and profits, do not stand in the same relation to the National Dividend.” (Marshall.)
    Which of these seems to you the sounder view?
  7. Explain what is meant by Consumer’s Rent; and examine the effect on Consumer’s Rent and on the aggregate satisfaction of the community, of a tax on a community subject to the law of Diminishing Returns.
  8. Explain the grounds which lead Professor Marshall to believe that the forces by which the wages of different grades of laborers are determined, work by a process similar to that by which the expenses of production determine the value of commodities.
  9. Examine carefully Professor Marshall’s view of the part played by rent of natural ability in determining manager’s earnings.
  10. Wherein is there similarity, wherein difference, in the positions of Carey and Bastiat in the history of economic theory?

Source: Harvard University Archives. Harvard University Examination Papers, 1873-1915. Box 3, Papers Set for Final Examinations in Philosophy, Political Economy, History, Roman Law, Fine Arts, and Music in Harvard College (June, 1892) in the bound volume: Examination Papers 1890-92.

Also found in Frank Taussig’s Scrapbook of his examinations. Posted earlier in Economics in the Rear-view Mirror.

_______________________

1891-92
POLITICAL ECONOMY 3.
Course Description and Enrollment

For Graduates and Undergraduates:—

[Political Economy] 3. Mr. [Edward] Cummings. — The Principles of Sociology. — Development of the Modern State, and of its Social Functions. 3 hours.

Total 25: 8 Graduates, 9 Seniors, 6 Juniors, 2 Others.

Source:   Harvard University, Annual Reports of the President and Treasurer of Harvard College, 1891-92, p. 54.

1891-92
POLITICAL ECONOMY 3.
Mid-year Examination, 1892.

[Not yet found]

 

1891-92
POLITICAL ECONOMY 3.
Final Examination, 1892.

[Arrange your answers in the order of the questions. Omit one.]

  1. “The liberty of the subject is only a means towards an end; hence, when it fails to produce the desired end, it may be set aside, and other means employed.”
    What do you conceive to be this “desired end”? On what is the prerogative in question founded?
  2. “When everything has been done to deter from crime or reform the criminal there will still remain a certain class whom it is hopeless to influence, and who must be dealt with in course of law, not for much result on themselves, but to carry out the principle of justice, and mainly to deter others.”
    Discuss the theoretical and practical validity of the principles of penal legislation here affirmed.
  3. “There are still two more weaknesses, which are peculiar to all states, not only to the modern elective State. From the strictly professional point of view, in the technical works which they direct; public functionaries have neither the stimulus nor the restraint of personal interest.”
    “Nearly every present acknowledged function of government has once been intrusted to private enterprise….Now, of all the enterprises which the state has thus appropriated to itself, there is not one which is not managed better and more wisely than it had been managed before by private parties. Most of them are such that the world has entirely forgotten that they were ever private enterprises.”
    What light is thrown on this controversy by the experience of continental governments in the management of railroads? Do the same arguments apply to railroads as to the telegraph, and the post? Why?
  4. State an criticize the theory of “surplus value.”
  5. “Let us suppose the whole field of industry covered by syndicates….Competition complained of by the Socialists would be largely gone, being merged within the syndicate; useless middlemen displaced; the employing capitalist with his too high wages replaced by a manager: all steps towards the Socialist goal. What is wanting chiefly?”
    Give a general outline of the Collectivist scheme, from the point of view of production, distribution and value.
  6. “But the bare labor-cost value, as it has been formulated up to now, invests the whole economy of socialism for the present with the character of a Utopia….It is remarkable, and even comforting, that all which is required to make socialism so much a matter of practical discussion, urges it to preserve, and even to intensify, the brighter elements of the liberal economic system.”
  7. “Moreover it is not difficult to deduce the necessity of State interference from Mr. Spencer’s own fundamental principles….The inspector is himself in fact, as Prof. Jevons says, a necessary product of social evolution and the division of labor.”
    Does expansion of public and municipal industry necessarily indicate the “gradual triumph of socialism”?
  8. Compare briefly the political and economic tendencies in Glasgow, London, and New York.
  9. What ground do you find for De Laveleye’s assertion that Socialism is pessimistic, while Political Economy is optimistic?
  10. State the arguments for and against municipal manufacture of gas in the United States.
  11. The systems of state education in the United States have been devised by the several states of the Union, and are exceedingly heterogeneous and defective. In certain States scarcely anything worthy of the name of education exists, while in others the systems have attained a high degree of perfection.”
    How in this respect does the United States compare with European countries.

Source: Harvard University Archives. Harvard University Examination Papers, 1873-1915. Box 3, Papers Set for Final Examinations in Philosophy, Political Economy, History, Roman Law, Fine Arts, and Music in Harvard College (June, 1892) in the bound volume: Examination Papers 1890-92.

_______________________

1891-92
POLITICAL ECONOMY 4.
Course Description and Enrollment

Primarily for Undergraduates:—

[Political Economy] 4. Mr. [William M.] Cole. — Economic History of Europe and America since the Seven Years’ War. — Lectures and written work. 3 hours.

Total 132: 35 Seniors, 40 Juniors, 40 Sophomores, 1 Freshman, 16 Others.

Source:   Harvard University, Annual Reports of the President and Treasurer of Harvard College, 1891-92, p. 54.

1891-92
POLITICAL ECONOMY 4.
Mid-year Examination, 1892.

[Not yet found]

 

1891-92
POLITICAL ECONOMY 4.
Final Examination, 1892.

[Answer both the questions in Roman numerals, and eight of the nine in Arabic numerals.]

  1. Compare the facilities for transportation by land as they existed in 1700 with those of 1830 and those of 1890.
    Do the same, for the same periods, for water transportation, for cotton manufacturing, and for banking.
  2. Compare the growth in the numbers of population in the United States since 1790 with the growth in the density of population per square mile. If you find any discrepancies, explain them.
  1. How far has England’s policy regarding free trade been affected by the policy of other nations?
  2. What, in your opinion, would have been the status in the United States of slavery, as a system of producing wealth, if emancipation had not taken place? State your reasons.
  3. Explain the change in the position of the American merchant marine at about the time of the Civil War.
  4. What influence has the extensive investment of capital in foreign countries upon the need, for the world’s commerce, of specie?
  5. By what process, and in what way, did the payment of the German indemnity by France affect the Crisis of 1873?
  6. How was the United States able to accumulate enough gold for Resumption in 1879?
  7. Was the indemnity demanded of France by Germany in 1871 just? Give your reasons.
  8. By what sort of processes has the United States reduced the annual burden of its debt faster than the principal?
  9. Why has England become the natural clearing-house for the world?

Source: Harvard University Archives. Harvard University Examination Papers, 1873-1915. Box 3, Papers Set for Final Examinations in Philosophy, Political Economy, History, Roman Law, Fine Arts, and Music in Harvard College (June, 1892) in the bound volume: Examination Papers 1890-92.

_______________________

1891-92
POLITICAL ECONOMY 5.
Course Description and Enrollment

For Graduates and Undergraduates:—

[Political Economy] 5. Professor [Frank W.] Taussig. — Railway Transportation. — Lectures and written work. 3 hours. 2d half-year.

Total 42: 3 Graduates, 22 Seniors, 10 Juniors, 1 Sophomore, 6 Others.

Source:   Harvard University, Annual Reports of the President and Treasurer of Harvard College, 1891-92, p. 54.

1891-92
POLITICAL ECONOMY 5.
Final Examination, 1892.

[Arrange your answers strictly in the order of the questions.]

  1. Compare the modes in which New York and Pennsylvania tried to secure communications with the West in 1825-1840.
  2. Sketch the salient events in the history of the New York Central Railway to the present time.
  3. Sketch the important provisions of the Thurman Act of 1878 in regard to the Pacific railroads, and the results which have ensued.
  4. Why was the railway mileage constructed in the United States in 1887 the largest yet reached?
  5. Why has the railway beaten the canal?
  6. Does the practice of charging what the traffic will bear result from the fact that railways present a case of industrial monopoly? Would it cease if competition were fully effective in railway operations?
  7. Discuss separately or together,
    (a) Whether the prohibition of railway pools is wise;
    (b) Whether there are grounds for permitting or prohibiting such combinations, which do not apply to attempts to bring about combination and monopoly in other industries.
  8. Point out wherein the schedules of maximum rates fixed by the State of Iowa resemble the German Reform Tariff, and wherein they differ from it.
  9. Explain what is the state of legislation as to long and short haul rates in the United States, England, France, and Germany; and state your opinion as to the desirability of preventing lower charges on the longer haul.
  10. Sketch the history of railway policy in Belgium.
  11. Why are railway pools and traffic agreements more stable in England than in the United States?
  12. Point out wherein the Railway Commission under the English Act of 1888 differs from the Interstate Commerce Commission.

Source: Harvard University Archives. Harvard University Examination Papers, 1873-1915. Box 3, Papers Set for Final Examinations in Philosophy, Political Economy, History, Roman Law, Fine Arts, and Music in Harvard College (June, 1892) in the bound volume: Examination Papers 1890-92.

_______________________

1891-92
POLITICAL ECONOMY 6.
Course Description and Enrollment

For Graduates and Undergraduates:—

[Political Economy] 6. Professor [Frank W.] Taussig. — History of Tariff Legislation in the United States. 3 hours. 1st half-year.

Total 64: 7 Graduates, 32 Seniors, 13 Juniors, 2 Sophomores, 10 Others.

 

Source:   Harvard University, Annual Reports of the President and Treasurer of Harvard College, 1891-92, p. 54.

 

1891-92
POLITICAL ECONOMY 6.
Mid-year Examination, 1892.

[Arrange your answers strictly in the order of the questions. Be concise. Answer all questions.]

  1. Criticize the arguments by which Hamilton endeavored to show (1) that agriculture was not more productive than manufactures; (2) that the greater division of labor and use of machinery in manufactures made the introduction of manufacturing industries peculiarly advantageous to a country.
  2. How do you explain the change, between 1820 and 1840, in the arguments as to the bearing of high wages on the protective system?
  3. Sketch the growth of the international trade of the United States from 1820 to 1860.
  4. Are there good grounds for saying that the tariff act of 1846 led to a period of general prosperity?
  5. In what way have the duties on fine woolens been higher in recent years than those on cheap woollens? Does the difference explain the fact that the domestic production is confined mainly to the cheaper goods? Give your reasons carefully.
  6. Explain the difference (1) in character, (2) in probable effects, between the Continental sugar bounties and the present United States bounty.
  7. Wherein would there probably be differences between the effects of reciprocity treaties (1) with Great Britain, admitting iron free; (2) with Great Britain, admitting wool from Australia free; (3) with Germany, admitting refined sugar free?
  8. How far is it true that the high level of wages in the United States is an effective obstacle to the successful prosecution of manufacturing industries?
  9. What were the duties on coffee, cotton goods, pig-iron, and wool, in 1799, 1819, 1839, 1859, 1879? (Use tabular form, if you wish.)
  10. How far did the South secure what it aimed at from the tariff act of 1833?
  11. Sketch the tariff legislation of 1872.
  12. Is it true that the adoption of a policy of free trade in England dates from the abolition of the corn-laws?

_______________________

1891-92
POLITICAL ECONOMY 7.
Course Description and Enrollment.

For Graduates and Undergraduates:—

[Political Economy] 7. Professor [Charles F.] Dunbar.

— First half-year:

The Theory and Methods of Taxation, with special references to local taxation in the United States. 3 hours.

Total 30: 2 Graduates, 20 Seniors, 8 Juniors.

— Second half-year

Banking, and the History of the leading Banking Systems. 3 hours.

Total 38: 2 Graduates, 22 Seniors, 10 Juniors, 1 Sophomore, 3 Others.

Source:   Harvard University, Annual Reports of the President and Treasurer of Harvard College, 1891-92, p. 54.

1891-92
POLITICAL ECONOMY 7.
Mid-year examination (first half-year), 1892.

[not yet found]

 

1891-92
POLITICAL ECONOMY 7.
Final examination (second half-year), 1892.

A.
Of these five questions one may be omitted.

  1. Which of the three great banks, the Bank of England, the Bank of France, and the Reichsbank, appears to you to present the best model for a great national bank, — and why?
  2. What peculiarities in the Scotch banking system account for the high credit and extended usefulness of the Scotch banks, and make their issue of £1 notes both necessary and safe?
  3. The value of a currency is said to depend on (a) its quantity, rapidity of circulation, and the amount of transactions to be effected, and (b) on the cost of the precious metals. How is this reasoning to be made applicable to deposits, considered as a part of the currency?
  4. A recent pamphlet contains the following:—
    “The ‘Currency Principle’ was advocated by Lord Overstone and others, and held that, under a system of free banking, over-issue [of convertible notes] is possible and likely to occur, inflating the currency. In England, the principle of limiting the issues was adopted in the Bank Act of 1844. A different application of the same principle obtains in this country under the National Bank system.”
    Discuss the closing statement in the above extract.
  5. As saving banks and banks of deposit and discount are alike bound to pay their depositors on demand, on what ground can investments be treated as safe or suitable for one of these classes of banks and not for the other? This may be illustrated by reference to investments in mortgages, in bank stock, and in commercial paper.

B.
Of these five questions one may be omitted.

  1. Describe Mr. Goschen’s proposals for increasing the stock of gold in the Bank of England and issuing £1 notes, and state the objects to be gained by the plan and the objections to it.
  2. Discuss the propositions, laid down by Mr. Buckner, in his speech of April, 1882, in opposition to the Bank Charters Extension Bill,—
    1. That the currency ought to be issued by the government;
    2. That an elastic currency is mischievous, as introducing an element of uncertainty, and that the government should therefore issue a fixed amount of convertible notes.
  3. It is urged that the characteristics which insure the high credit and universal currency of the national bank circulation,—
    “are qualities which help to make its movements unnatural, artificial, and impart to it a roaming character, helping to force it away from the issuer, away from the country districts where it is needed, and consequently to induce its accumulation when out of active commercial employment in the great financial centres, and while there to foster and become more or less fixed in speculative ventures—that is, unresponsive to commercial influences when needed for commercial work.”
    Discuss the question whether issues having only local credit would remedy the difficulties suggested above?
  4. Discuss the following proposition for the issue of bank-notes under State authority:—
    1. Take off the present 10 per cent. tax from the notes of any bank complying with the following regulations:—
    2. Permit any State to tax circulation, in order to accumulate a fund to redeem notes of such of its own banks a may fail.
    3. Forbid any bank to issue notes in excess of two-thirds of its capital.
    4. Make notes a first lien on all assets of the issuing bank.
    5. Require coin redemption by the banks and a coin reserve of 25 per cent. of outstanding notes.
    6. Leave any State free to forbid or permit the issue of notes under the above regulations by banks within its jurisdiction.
      [Commercial and Financial Chronicle, May 14.]
  5. Discuss the following propositions for completely free banking, made by Courcelle-Seneuil (Traité des Opérations de Banque, Book IV., ch. ix., §3):—
    “Il vaudrait mieux donner au premier venu le droit d’émettre des billets à vue et au porteur sous certaines conditions définies par la loi….On doit supposer que le banquier sait mieux son métier que le législateur; celui-ci ne doit point réglementer ce qui est du métier; il doit se borner à prévenir la fraude, et il ne peut mieux y parvenir qu’en imposant au banquier un fort cautionnement envers le public, c’est-à-dire un fort capital….Les vérifications officielles de portefeuille ne peuvent présenter au public aucune garantie.”

Source: Harvard University Archives. Harvard University Examination Papers, 1873-1915. Box 3, Papers Set for Final Examinations in Philosophy, Political Economy, History, Roman Law, Fine Arts, and Music in Harvard College (June, 1892) in the bound volume: Examination Papers 1890-92.

_______________________

1891-92
POLITICAL ECONOMY 8.
Course Description and Enrollment.

For Graduates and Undergraduates:—

[Political Economy] 8. Professor [Charles F.] Dunbar. — History of Financial Legislation in the United States. 3 hours. 2d half-year

Total 50: 7 Graduates, 22 Seniors, 11 Juniors, 2 Sophomores, 8 Others.

Source:   Harvard University, Annual Reports of the President and Treasurer of Harvard College, 1891-92, p. 54.

1891-92
POLITICAL ECONOMY 8.
Final Examination, 1892.

Two questions may be omitted.

  1. What were the terms on which the different portions of the revolutionary debt were made redeemable by the act of August 4, 1790, and when and how was their redemption actually undertaken?
  2. What are the leading cases of suspension of specie payments in the United States since 1789, and what were the general causes in each case?
  3. What was the method by which specie payments were resumed in 1817?
  4. Von Holst says (II., p. 32), “Jackson did not come to Washington resolved to wipe out the bank.” What is probably the truth as to Jackson’s attitude towards the bank when he was inaugurated, and as to the breaking out of the bank war?
  5. What were the “branch drafts” issued by the branches of the second United States Bank, the reasons for their issue, and the objections thereto?
  6. What is the history of the following item in the general account of the Treasurer of the United States:—
    “Unavailable amount on deposit with the States, $28, 101,645.”
  7. What were Mr. Chase’s reasons for urging the establishment of the national banking system?
  8. How does the legal tender decision in Juillard vs. Greenman differ in principle from that in the earlier case of Knox vs. Lee?
  9. What were Mr. McCulloch’s reasons for wishing to establish the policy of contracting the currency without delay in 1865?
  10. What influences led Congress to restrict, and finally annul, Secretary McCulloch’s authority for retiring United States notes? Give approximate dates of the Acts.
  11. President Grant wrote, in 1874:—
    “I would like to see a provision that…the currency issued by the United States should be redeemed in coin…and that all currency so redeemed should be cancelled and never be re-issued.” [ To Jones.]
    How does this compare with the redemption actually practiced under the Resumption Act, and how came the present practice to be adopted?
  12. Sherman, speaking of the first Legal Tender Act, said:—
    “We agreed in that act that we would apply one per cent. of the principal of the debt to the payment of the debt. The debt is now $2,5000,000,000. One per cent. is $25,000,000, and that must not only be applied every year, but it must be applied in the nature of a sinking fund.” [Speeches p. 264.]
    How far has the government followed this interpretation of the act?

Source: Harvard University Archives. Harvard University Examination Papers, 1873-1915. Box 3, Papers Set for Final Examinations in Philosophy, Political Economy, History, Roman Law, Fine Arts, and Music in Harvard College (June, 1892) in the bound volume: Examination Papers 1890-92.

Categories
Economists Harvard

Harvard. Life of accounting professor William Morse Cole, A.M. 1896

 

In preparing the previous post, “Harvard final exams in political economy and ethics of social reform, 1889-1890“, I saw that the Harvard Business School accounting professor, William Morse Cole, got his professional start in the department of political economy as Frank Taussig’s side-kick for Political Economy 1, the principles course of its time. Answering the call of due diligence, I decided to put together a biographical post on the man. To be honest, I did not feel at all inspired about the prospect of checking out the career of this early Harvard Business School professor. While I confess to loving national income and product accounts (and especially deflating them with theoretically appropriate price indexes), I’ve never warmed to the nuts-and-bolts of actual economic accounting. Literally, go figure. Still, I overcame my nasty prejudice enough to track down Professor Cole and so we “Meet a Harvard A.M. alumnus…”

Cole wrote a battery of accounting textbooks that one could logically expect from a professor of accounting. These are listed below with links. Three other books by Cole were definitely composed outside his accounting lane. Cole’s first book was a romance written under a pseudonym, his mid-life tome “An American Hope” offered his readers a series of meaning-of-life reflections, and towards the end of his career he even published a high-school elementary economics textbook (encouraged by no less a mover-shaker in the economics profession than Richard Ely).

 

_________________________

Harvard University, in memoriam

WILLIAM MORSE COLE, Professor of Accounting, Emeritus, died December 15, 1960, in his ninety-fifth year. A graduate of Harvard College in 1890, Mr. Cole first offered a non-credit course in accounting in the Department of Economics in 1900-01 as a vocational aid for seniors. Five years later the course achieved credit status, and when the Business School was founded in 1908, Mr. Cole became an Assistant Professor there and teacher of one of the first required courses. He was made a full Professor in 1916 and continued to teach accounting until his retirement in 1933. A highly respected teacher known for his strict intellectual discipline, he was the originator of one of the major types of accounting statements, the “source and application of funds statement,” used extensively by professional accountants, and his books on accounting were highly regarded. A gracious and thoughtful person, he continued alert and interested in Harvard affairs until his death.

Source: Harvard University. Report of the President of Harvard College, 1959-1960, p. 26.

_________________________

Life of William Morse Cole in Dates

1866. Born February 10 in Boston, Massachusetts.

Family moved to Portland, Maine. High school and Portland Business College.

1890. A.B. Harvard College

1890-93. Instructor in political economy, Harvard College.

1894-95. Secretary, Massachusetts Commission on Unemployed.

1895-98. Harvard University Extension Lecturer.

1896. A.M. Harvard.

1898-1901. High school teacher of English literature and composition in Fall River, Massachusetts.

1900-08. Harvard Instructor in Principles of Accounting.

1901-08. High School teacher of English literature and composition at South High School, Worcester, Massachusetts.

1908-13. Assistant Professor, Harvard Business School.

1913-16. Associate Professor, Harvard Business School.

1917-1919. Served twenty months as captain in the Quartermaster Corps.

1916-1933. Professor of Accounting, Harvard Business School.

1931. Visiting Instructor at University of California, Berkeley Summer Session.

1933. Professor Emeritus of Accounting, Harvard.

1960. Died December 15 at age 94 in New Britain, Connecticut.

Sources:

William Morse Cole” in John J. Kahle, American Accountants and Their Contributions to Accounting Thought, (Reprinted Routledge).
University of California. Intersession and Summer Session 1931 at Berkeley (1931), p. 4.
Harvard Business School Yearbook 1924-25, p. 13.
Harvard University. Report of the President of Harvard College, 1959-1960, p. 26.

_________________________

Life of William Morse Cole in Books

1895. An Old Man’s Romance, a Tale. (Pseudonym “Christopher Craigie”). Boston: Copeland and Day.

1908. Accounts. Their Construction and Interpretation for Business Men and Students of Affairs. Boston: Houghton Mifflin Company.

“The first issue of this book was brought out at a time when no general, non -technical, non-professional treatise on accounting had been published . The author had then been giving for eight years a course of instruction to seniors in Harvard College on the principles of accounting, and believed that many business men and students of affairs would be interested to see briefly but comprehensively how accounts are constructed and interpreted.”

Revised and enlarged edition, 1915.

1910. Accounting and Auditing. Minneapolis: Cree Publishing Company.

1910. The American Hope. New York: D. Appleton.

“The academic point of view is apt to be out of joint with real life and may be said to be a disease to be gotten rid of at all hazards, but once in a while from academic circles comes forth a book filled with a knowledge of both the real and the ideal. The study of the use of the possessive by Shakspeare or the dative by Virgil has been supplanted by study of life understood and depicted so well by both Shakspeare and Virgil and the result is a book of real worth, as in this book on the American Hope by a Harvard teacher.

‘The fundamental ground of American hope,’ says Mr. Cole, ‘is the prevailing idealism of the American character.’ Not money or things in themselves, but power, life, and the ideal are behind our material development. Business on a large scale is the child of the ideal. The chapter on the power of choice is a pretty exposition of the responsibility of the irresponsible, and, like all essays concerning the freedom of the will, finds an escape from both heredity and surroundings by the pursuit of truth and life. The marriage tie is a great spring of progress and the ideal is brought out quite strongly. ‘It is doubtless true that if the only marriages contracted were those classed in this chapter as perfect, the marriage rate would decline rapidly. This might not be a calamity. Few parts of the world are today suffering from lack of population. What the world needs is not at all more people, but more people begotten and trained In ideal conditions.’

The Training of Powers, The Fraternal Bond, The Still, Small Voice, The Will of the Community and the Attitude Toward Life are the titles of various chapters and a reflection of the character of the book. Economic Freedom is discussed in a separate chapter and the root of progress is clearly brought out. The pressure of population on subsistence, thrift, sobriety, and industry are discussed. Civilisation in every one of its aspects is a struggle against the animal instincts.’ The book is a thoroughly readable one from beginning to end and it is not too high praise to say that it is a rare one.”  Boston Evening Transcript, 22 June 1910, p. 22.

1913. Cost Accounting for Institutions. New York: Ronald Press Company.

1914. Bookkeeping, Accounting and Auditing. Racine: National Institute of Business.

1915. Problems in the Principles of Accounting. Cambridge, Massachusetts: Harvard University Press.

“These problems have been designed for use with the author’s Accounts: Their Construction and Interpretation, published by Houghton Mifflin Company, of Boston.”

1921. The Fundamentals of Accounting. With the collaboration of Anne Elizabeth Geddes, A.B. Boston: Houghton Mifflin.

1926. Economic Success. New York: Macmillan.

“This book is an attempt to present fundamental economic principles in form and content comprehensible by young people of the age of the highest grades in elementary and intermediate schools.” Thanks Richard T. Ely “for the original suggestion that this book be written, and for helpful criticism both of its proposed content and of the form which the manuscript took.” (p. xii).

Image Source: Harvard Class Album 1916.

Categories
Exam Questions Harvard

Harvard. Exams for Political Economy Courses and Ethics of Social Questions, 1890-1891

 

With the academic year 1890-91, a new instructor joined the Harvard political economy team, William Morse Cole who co-taught Political Economy 1 with Frank Taussig and  Political Economy 8, History of Financial Legislation in the United States. Cole went on to have a successful career as professor of accounting at Harvard Business School. 

The previous year’s exams have been transcribed and posted earlier:

Harvard. Final exams in political economy and ethics of social reform, 1889-1890

_________________________

1890-1891. Philosophy 14.

Enrollment

[Philosophy] 11. Professor F. G. Peabody. — The Ethics of Social Questions. — The questions of Charity, Divorce, the Indians, Temperance, and the various phases of the Labor Question (Socialism, Communism, Arbitration, Cooperation, etc.), as questions of practical Ethics. — Lectures, essays, and practical observations. 3 hours.

Total 103: 4 Graduates, 53 Senior, 28 Juniors, 4 Sophomores, 3 Freshmen, 11 Others.

Source: Harvard University. Annual Report of the President of Harvard College 1890-1891, p. 57.

 

 

PHILOSOPHY 14.
THE ETHICS OF THE SOCIAL QUESTIONS.
Mid-year examination (1890-91)

[Omit one question.]

  1. The attitude of political economy toward the questions of social reform.
  2. Consider the possible relations in which your own life may stand to the life of society, and their ethical significance.
  3. Examine a special case of moral heroism and state what, in your opinion, was its motive.
  4. What, according to Professor Sumner, are the duties toward others of “a free man in a free democracy”? Why? And with what result, in your opinion, to society?
  5. Enumerate and illustrate some of the practical rules of good charity which issue from the philosophy of charity.
  6. The relation of the labor question in France and England to the political history of those countries.
  7. Compare the conditions of wealth and the possibilities of revolution in England and in this country.
  8. Consider Carlyle’s doctrine of the “Captain of Industry” as a solution of the modern labor question.
  9. What does Ruskin mean by: Roots of honor; veins of purple; non-competitive just exchange; ad valorem?
  10. What is there in religion which encourages the Socialist and what is there which repels him, and what relation between socialism and religion is, in your judgment, likely to be the result?

 

Source: Harvard University Archives. Harvard University, Mid-year examinations, 1852-1943. Box 2. From bound volume Examination Papers, 1890-91.

 

 

 

PHILOSOPHY 14.
THE ETHICS OF THE SOCIAL QUESTIONS.
Year-end examination (June 1891)

[Omit one question.]

  1. “All wealth is due to labor; therefore to labor all wealth is due.” What, in your opinion, is the economic importance, and the justice, of this proposition?
  2. Compare the views of the Socialist, the Individualist and the “Opportunist” as to the tendency toward State interference. What is your own view of the merits of this kind of legislation?
  3. “Well may Prince Bismarck display leanings toward State Socialism.” Why does Mr. Spencer make this remark, and with what justice?
  4. On what principle would a Professor be paid, under the Socialist programme?
  5. Consider the commercial advantages and hindrances of an establishment like the Hebden Bridge Fustian Mill.
  6. Compare the principle of profit-sharing formerly used in the American Fisheries with that represented by the firm of Billon et Isaac.
  7. Describe the general features and the main intention of the Dawes Indian Bill; and the supplementary legislation now proposed.
  8. What is meant by the “Philosophy of the Family,” and what is its relation to the modern Divorce Question?
  9. If you should enter the retail liquor business in Boston, what legal restrictions would you find hampering the freedom of your trade?
  10. Apply the doctrine of the “Social Organism” to the question of Temperance.

 

Source: Harvard University Archives. Harvard University, Examination papers, 1873-1915. Box 3.  Papers Set for Final examinations in Philosophy, Political Economy, History, Roman Law, Fine Arts, and Music in Harvard College (June, 1891) in bound volume Examination Papers, Mid-Year 1890-91.

_____________________________

1890-91
POLITICAL ECONOMY 1.

Enrollment.

[Political Economy] 1. Professor [Frank W.] Taussig and Mr. [William Morse] Cole.

First half-year: —

Mill’s Principles of Political Economy. 3 hours

Second half-year: —

Division A (Theoretical): Mill’s Principles of Political Economy. — Cairnes’s Leading Principles of Political Economy. 3 hours.

Division B (Descriptive): Lectures on Finance, Labor and Capital, Coöperation. — Hadley’s Railroad Transportation. —  Laughlin’s Bimetallism. 3 hours.

Total 201:

A: 10 Seniors, 53 Juniors, 50 Sophomores, 2 Freshmen, 19 Others.
B: 12 Seniors, 31 Juniors, 15 Sophomores, 2 Freshmen, 7 Others.

Source: Harvard University. Annual Report of the President of Harvard College 1890-1891, p. 58.

 

Student notes available at the Harvard Archives

Kennedy, Frank Lowell, Notes on lectures by Frank W. Taussig and William M. Cole on Political Economy 1, 1890-1891. Harvard University Archives HUC 8890.371.1

 

1890-91.
POLITICAL ECONOMY 1.
Mid-year examination.

[Divide your time equally between the two parts of the paper.]

I.
[Omit two.]

  1. “Whether men like it or not, the unproductive expenditure of individuals will pro tanto tend to impoverish the community, and only their productive expenditure will enrich it.”
    “It would be a great error to regret the large proportion of the annual produce which in an opulent country goes to supply unproductive consumption.”
    Can you reconcile these two statements of Mill’s?
  2. “Hardly any two dealers in the same trade, even if their commodities are equally good and equally cheap, carry on their business at the same expense, or turn over their capital in the same time. That equal capitals give equal profits, as a general maxim of trade, would be as false as that equal age or size give equal bodily strength, or that equal reading or experience give equal knowledge.” Can you reconcile this statement of Mill’s with the doctrine of the tendency of profits to an equality?
  3. How far is it true that a general rise or fall in wages would not affect values?
  4. Suppose a country having a metallic currency to issue inconvertible paper to one-half the amount of the coin, and trace the effects on prices and on the circulating medium (1) in an isolated country, having no international trade; (2) in a country having international trade.
  5. On the same supposition, trace the effects, in the country having international trade, on the foreign exchanges, on the course of international trade, and on the terms of international exchange.
  6. “If consumers were to save and covert into capital more than a limited portion of their income, and were not to devote to unproductive consumption an amount of means bearing a certain ratio to the capital of the country, the extra accumulation would be merely so much waste, since there would be no market for the commodities which the capital so created would produce.” Is this true?

II.
[Answer all.]

  1. “Capital is not the result of saving; it is not an accumulation. Its nature is that it should be consumed almost as fast as it is produced. … Saving or accumulation would necessarily defeat the end of its existence. How can materials or tools be saved?” Answer the question.
  2. Explain why rent is not an element in the cost of production of the commodity which yields it.
  3. Connect the law of the increase of labor with the law of production from land.
  4. What is the effect of gratuitous education for a profession on the wages of those engaged in it?
  5. Why does the durability of the precious metals give stability to their value?
  6. What are the laws of value applicable to (1) iron ore, (2) watch-springs, (3) wool and mutton, (4) patented bicycles?
  7. How does the rate of interest bear on the price of land and of securities?

Source: Harvard University Archives. Harvard University, Mid-year examinations, 1852-1943. Box 2. From bound volume Examination Papers, 1890-91.

 

POLITICAL ECONOMY 1.
Year-end examination (June 1891)
Division A.

Arrange your answers strictly in the order of the questions.

  1. Wherein is the effect of a change in the demand for commodities on the wages-fund different (1) if competition among laborers is effective? (2) if it is not effective?
  2. On what grounds does Cairnes reach the same conclusions, as to the possible effects of Trades Unions on general wages, for England and for the United States?
  3. Examine Cairnes’s reasoning as to the possibility of maintaining the accumulation of capital in a socialist community.
  4. Why are the wages of women, according to Mill, lower than the wages of men? Accepting Mill’s explanation, what would Cairnes say as to the laws of value applicable to the exchange of the products of women’s labor with the products of men’s labor?
  5. What is the error in saying that high wages make high prices?
  6. “Gold may be cheap, and prices at the same time be low.” Explain.
  7. Is it true that the benefit of foreign trade lies in its affording an outlet for the surplus produce of the community?
  8. Suppose the people of the United States to borrow annually large sums from Europe; and suppose them also to have large interest payments to make on loans contracted in previous years; would you expect our foreign trade to show an excess of imports or of exports?
  9. Mill says that an emission of inconvertible paper money, equal in amount to the specie previously circulating, will drive out the whole of the metallic money; “that is, if paper be issued of as low a denomination as the lowest coin; if not, as much will remain as convenience requires for the smaller payments.” What light is thrown on this statement by the experience of the United States in 1862?
  10. Why did the circumstance that an exceptionally large part of the country’s business was done for cash in the period immediately after the civil war make the time favorable for a speedy contraction of the currency?
  11. Is payment for capital sunk in the soil, rent, or profit?

Division B.

[Arrange your answers strictly in the order of the questions.]
[One question may be omitted.]

  1. Why have pools and traffic agreements been more stable in England than in the United States?
  2. Why does a railroad charge more per ton per mile on cotton goods than on coal?
  3. “A government enterprise may be managed on any one of four principles: 1. as a tax; 2. for business profits; 3. to pay expenses; 4. for public service, without much regard to the question of expenses.” Explain, giving an example under each head.
  4. Wherein is there a resemblance between the legislation of France as to railways and as to banking?
  5. Arrange in proper order the following items of a bank account: Capital, 300; Loans, 1150; Bonds and Stocks, 50; Surplus, 85; Undivided Profits, 10; Cash, 110; Cash items, 90; Notes, 90; Real Estate, 25; Other Assets, 20; Deposits, 960.
    Do you see any reason for believing this bank to be or not to be a national bank of the United States? To be a city or a country bank?
  6. Suppose the national banks of the United States ceased to issue notes, their other operations remaining as now; how great would be the effect of the change on the circulating medium of the community? Compare the effect with that which would ensue in Germany if the note issue of the German banks were to cease.
  7. Under the national bank act, how does the action of our banks, when their reserves are suddenly lowered, differ from that of the Bank of England in like case?
  8. Wherein is the mode of dividing profits among members of the coöperative stores in England different from that of the coöperative building associations of the United States?
  9. Wherein does bimetallism as now practiced in France differ from bimetallism as it was in France in 1850? Wherein does it differ from bimetallism as it is in the United States now?
  10. Compare the legislation of Germany on coinage in 1873 with that of the United States in 1853.
  11. Mill divides commodities into three classes, according to the laws of value applicable to them. In which class would you put silver bullion?
  12. Describe carefully the act for the resumption of specie payments, stating when it was passed, when it went into effect, and how far it was successful in accomplishing the desired object.
  13. Will a general rise in wages affect values? prices? profits?

 

Source: Harvard University Archives. Harvard University, Examination papers, 1873-1915. Box 3. Papers Set for Final examinations in Philosophy, Political Economy, History, Roman Law, Fine Arts, and Music in Harvard College (June, 1891) in bound volume Examination Papers, 1890-92.

_____________________________

1890-91
POLITICAL ECONOMY 2.

Enrollment.

[Political Economy] 2. Professor [Frank W.] Taussig and Mr. [John Graham] Brooks. — History of Economic Theory. — Examination of selections from Leading Writers. — Socialism. 3 hours.

Total 23: 4 Graduates, 10 Seniors, 8 Juniors, 1 Other.

Source: Harvard University. Annual Report of the President of Harvard College 1890-1891, p. 58.

 

Previously Posted

https://www.irwincollier.com/harvard-history-of-economic-theory-final-exam-questions-taussig-1891-94/

Fun fact: W.E.B. Dubois was enrolled in Economics 2 in 1890/91 as a graduate student and was awarded a grade of A (one of six awarded to the twenty-two who received grades,  as recorded in Taussig’s scrapbook).

1890-91.
POLITICAL ECONOMY 2.
Mid-year examination.

[Divide your time equally between the two parts of the paper.]
[Omit one question.]

  1. It has been suggested that the real source from which wages are paid is not the product of the laborer, nor the capital of the employer, but the income of the consumer. What should you say?
  2. To which of the following cases, if to any, is the reasoning of the wages fund theory applicable? (1) The farmer tilling his own land with his own capital; (2) the fisherman working for a share of the catch; (3) the independent artisan working on his own account with borrowed capital; (4) the employer who habitually sells his product before pay-day, and pays his laborers with the proceeds.
  3. Wherein does President Walker’s view of the source from which wages are paid differ from George’s?
  4. “The extra gains which any producer or dealer obtains through superior talent in business, or superior business arrangements, are very much of a similar kind [to rent.] If all his competitors had the same advantages, and used them, the benefit would be transferred to the consumers, through the diminished value of the article; he only retains it for himself because he is able to bring his commodity to market at a lower cost, while its value is determined by a higher…. Wages and profits represent the universal elements in production, while rent may be taken to represent the differential and peculiar; any difference in favor of certain producers, or in favor of production in certain circumstances, being the source of a gain, which, though not called rent, unless paid periodically by one person to another, is governed by laws entirely the same with it.”—Mill, Political Economy, book iii., ch. v., §4.
    What has President Walker added to this in his discussion of business profits?
  5. “It is true that money does not beget money; but capital does manifestly beget capital. If a man borrows a thousand ducats and ties them up in a bag, he will not find any little ducats in the bag at the end of the year; but if he purchases with the ducats a flock of sheep, he will, with proper attention, have lambs enough at the end of the year to make a handsome interest on the loan, and make a handsome profit for himself. If the turns the ducats into corn, he will find it bringing forth, some thirty, some sixty, some an hundred fold…Very seldom does a man borrow money to use it, as money, through the term of his loan. When he does so, as brokers for example sometimes do, he may to Antonio’s question, ‘Is your gold and silver rams and ewes?’ return Shylock’s answer, ‘I cannot tell; I make it breed as fast.’”
    Discuss this explanation of interest. Whom do you suppose to be the writer of the extract?
  6. “The natural history of the notion on which it [the wages-fund doctrine] rests, is not obscure. It grew out of the conditions which existed in England during and immediately subsequent to the Napoleonic wars. Two things were then noted. First, capital had become accumulated in the island to such an extent that employers found no (financial) difficulty in paying their laborers by the month, week, or day, instead of requiring them to await the fruition of their labor in the harvested or marketed product. Second, the wages were, in fact, generally so low that they furnished no more than a bare subsistence, while the employment offered was so restricted that an increase in the number of laborers had the effect to throw some out of employment or to reduce the wages for all. Out of these things the wages-fund theory was put together.”
    Examine this account of the rise of the wages-fund doctrine.
  7. Discuss the method of reasoning followed by Adam Smith, and illustrate by his treatment of two of the following topics: (1) the causes which bring about high wages; (2) the effects on domestic industry of restraints on importation; (3) the origin and effects of the division of labor.
  8. Explain how Ricardo’s conception of wages bears on his conclusions as to the effects of taxes on wages, and as to the net income of society.
  9. What can be said in justification of the views of the writers of the mercantile school?

Source: Harvard University Archives. In Harvard University, Mid-year examinations, 1852-1943. Box 2. From bound volume Examination Papers, 1890-91.

 

Second semester taught by John Graham Brooks.

POLITICAL ECONOMY 2.
Year-end examination (June 1891)

  1. From Rousseau to the Fabians, what have been the chief historic changes in the Philosophy of Socialism?
  2. In detail, state the differences between the Marx type of Socialism and that of the Fabians.
  3. With reference to the “three rents” what are the most important objections to Socialism?
  4. What reasons can you give to show that Socialism is likely to have much further development in our society?

Source: Harvard University Archives. Harvard University, Examination papers, 1873-1915. Box 3. Papers Set for Final examinations in Philosophy, Political Economy, History, Roman Law, Fine Arts, and Music in Harvard College (June, 1891) in bound volume Examination Papers, 1890-92.

_____________________________

1890-91
POLITICAL ECONOMY 3.

Enrollment.

[Political Economy] 3. Mr. [John Graham] Brooks. Investigation and Discussion of Practical Economic Questions. — Social Questions. — Short theses. 1st half-year. 3 hours.

Total 10: 1 Graduate, 7 Seniors, 1 Junior, 1 Other.

Source: Harvard University. Annual Report of the President of Harvard College 1890-1891, p. 58.

 

1890-91
POLITICAL ECONOMY 3.
Mid-year examination

  1. State the general objects of the German State Insurance, with reasons why it is likely, or not likely, to reach its objects.
  2. State in detail the strong points and the weak points of Trades Unions.
  3. What is the effect of Trades Unions upon their own wages, as distinguished from wages in general?
  4. What advantages has Profit Sharing over the present forms of the wages system?
  5. Are there reasons to believe that Profit Sharing will have much larger influence in the future?
  6. With special reference to the work of the half year, what “social remedies” appear to you most promising?

Source: Harvard University Archives. In Harvard University, Mid-year examinations, 1852-1943. Box 2. From bound volume Examination Papers, 1890-91.

_____________________________

1890-91
POLITICAL ECONOMY 4.

Enrollment.

[Political Economy] 4. Professor [Charles Franklin] Dunbar. — Economic History of Europe and America since the Seven Years’ War. —Lectures and written work. 3 hours.

Total 103: 29 Seniors, 28 Juniors, 25 Sophomores, 4 Freshmen, 17 Others.

Source: Harvard University. Annual Report of the President of Harvard College 1890-1891, p. 58.

Other course material available at the Harvard Archives

Topics and references in political economy IV [1891?]. Student’s copy belonging to C. King Morrison, ’91. with manuscript notes. Harvard University Archives HUC 8890.371

Larrabee, Ralph Clinton (A.B. 1893). Notes in Political Economy 4: lectures by Prof. Dunbar, 1890-1891. Harvard University Archives HUC 8890.371.4.48

Principle text for Political Economy 4

From the prefatory note to Benjamin Rand’s (ed.) Selections illustrating economic history since the Seven Years’ War (Cambridge, MA: Waterman and Amee, 1889):

These selections have been made for use as a text-book of required reading to accompany a course of lectures on economic history given at Harvard College.

1890-91
POLITICAL ECONOMY 4.
Mid-year examination

Lay out your time carefully, reserving 15 minutes for review and correction.

A.
Give half of your time (say 80 minutes) to A, omitting one question.

  1. Make a careful statement of the leading provisions of the English navigation and colonial system.
  2. Adam Smith’s reasons for saying that the policy of Great Britain towards her colonies had, upon the whole, been less illiberal and oppressive than that of other countries towards theirs. [Rand, pp. 12-26.]
  3. Contrast the effects of the French revolutionary period upon the holding and distribution of land, in France and Germany respectively.
  4. Describe the current of opinion and the industrial conditions which made free trade inevitably the policy for England.
  5. State the reasons for the logical and political importance of the corn laws in the English free trade movement.
  6. The essential differences between the French and the Anglo-American methods of managing railway construction and ownership, and the effect and advantages of each.

B.
Give 80 minutes to B, omitting two questions.

  1. The trade between the United States and the British West Indies, before our revolution, and after.
  2. The inventions or improvements which made the development of the cotton States possible.
  3. The condition of commerce and manufactures in the United States in the two periods, 1794-1808, and 1808-1815.
  4. The successive enterprises for opening communication with the territory north of the Ohio, and their importance.
  5. The English legislation respecting cotton goods in the last century and the reasons for it.
  6. What was the effect of the Napoleonic wars upon the introduction of manufactures on the continent of Europe?
  7. The comparative state of preparation of England, France, Germany, and the United States for undertaking the modern industries when the peace of 1815 came.
  8. Give what account you can of the career and opinions of Turgot, with dates.
  9. The contributions of Stein and Hardenberg respectively to the reform in the Prussian system of land-holding.
  10. On what plan was the Zollverein organized?
  11. What are the differences in industrial characteristics which make it natural for England and France to adopt different policies as to protection and free trade?

Source: Harvard University Archives. In Harvard University, Mid-year examinations, 1852-1943. Box 2. From bound volume Examination Papers, 1890-91.

 

1890-91
POLITICAL ECONOMY 4.
Year-end examination (June 1891)

Lay out your time carefully, reserving 15 minutes for review and correction

A.
Give half of your time to A. omitting one question.

  1. The establishment of the Zollverein is spoken of [Rand, page 138] as “the first step towards what is called the Germanization of the people” and to have “prepared the way for a political nationality.” Show how it had this effect.
  2. Cairnes, in his discussion of the new gold, [Rand, page 197] shows that “a given addition to the metallic stock of Great Britain and the United States … will cause a greater expansion of the total circulation, and therefore will support a greater advance in general prices, that the same addition to the currency of … France … and that again, the effect in countries like France will be greater than in countries like India or China.” Why is this, and how much effect did this difference in sensitiveness produce in the years after 1850?
  3. Cairnes lays down [Rand, page 209] that “every country is interested in raising as rapidly as possible the prices of its productions,–in other words, in the most rapid possible depreciation in the local value of its gold.” What are the grounds for this proposition?
  4. The writer in Blackwood’s, [Rand, page 228] says that the most important point in the payment of the French indemnity is “How came it that £170,000,000 [4,250,000,000 francs] of bills could be got at all”? What is the explanation of this fact?
  5. What are the reasons for looking upon Italy as a possible serious competitor in ocean navigation, and what are her great drawbacks in such competition?

B.

  1. What are the marked differences between the great change in the production and value of the precious metals in the sixteenth century, and that in the nineteenth?
  2. Why was an additional supply of gold especially important to the world in the years 1850-60?
  3. How does the mere saving of time in transportation, or in the transmission of intelligence, produce an effect upon commerce?
  4. How far was the civil war the cause of the decline of American shipping after 1860?
  5. What reasons made the breaking out of a financial crisis in the United States in 1873 easier than usual?
  6. Why did the payment of the French indemnity disturb the financial quiet of other countries than France and Germany?
  7. What are the great cases of resumption of specie payment in the years 1875-85, and how were they brought about respectively?
  8. Why is the trade between countries so often “triangular,” and why is England so generally one of the parties concerned?
  9. How does the modern theory of the utility of colonies differ from that of a century or two ago?

Source: Harvard University Archives. Harvard University, Examination papers, 1873-1915. Box 3. Papers Set for Final examinations in Philosophy, Political Economy, History, Roman Law, Fine Arts, and Music in Harvard College (June, 1891) in bound volume Examination Papers, 1890-92.

_____________________________

1890-91
POLITICAL ECONOMY 6.

Enrollment.

[Political Economy] 6. Professor [Frank W.] Taussig. — History of Tariff Legislation in the United States. 2 hours. 2d half-year

Total 43: 1 Graduate, 28 Seniors, 12 Juniors, 2 Others.

Source: Harvard University. Annual Report of the President of Harvard College 1890-1891, p. 58.

 

 

1890-91
POLITICAL ECONOMY 6.
Year-end examination (June 1891)

[Arrange your answers strictly in the order of the questions. Answer all questions.]

  1. Explain wherein France and the United States were in similar positions, as regards customs policy, in 1814-15; and state briefly the legislation to which these situations led in the two countries.
  2. What was the argument, discussed in Hamilton’s Report on Manufactures, which rested on the supposed exceptions productiveness of agriculture? What was Hamilton’s answer? What is the sound view?
  3. How did Gallatin propose in 1831 to fix customs duties without regard to their protective effect? Walker in 1845?
  4. Was there any ground in 1832 for saying that the duties on imports were equivalent to duties on exports? Is there now?
  5. How did the general fall in prices after 1819 affect the growth of manufactures in the United States?
  6. “The climate, soil, and conditions generally in the Northwest, are very favorable to the cultivation of flax fibre as well as of the seed. After a short experience as to the primary manipulation and handling of the flax fibre, our farmers would produce flax which would compare favorably with the best varieties of the fibre. It seems strange that a practical people like ourselves should for years have been satisfied to cultivate seed for flax at a value of about $15 per acre, and at the same time allow 600 pounds of flax fibre per acre to rot on the ground, this fibre having a value, after being manipulated, of $186 per ton.”
    Can you explain the anomaly?
  7. Describe the process by which the duties on woollen cloths, as they stand in the act of 1890, were arrived at.
  8. What ground is there for saying that the protective movement in the United States is part of a general reaction towards protection which has appeared in most civilized countries in recent years?
  9. It has been said that the tariff act of 1789 began the protective policy of the United States; that the act of 1816 was the first giving serious protection; that the act of 1824 was the first strictly protective act. Which statement, if any, do you think true?
  10. Which of the important tariff acts between 1835 and 1859 were passed quite without regard to financial considerations?
  11. What does the continued importation of clothing wool indicate as to the effect of the duty on the domestic price? of pig iron? of silks?

Source: Harvard University Archives. Harvard University, Examination papers, 1873-1915. Box 3. Papers Set for Final examinations in Philosophy, Political Economy, History, Roman Law, Fine Arts, and Music in Harvard College (June, 1891) in bound volume Examination Papers, 1890-92.

_____________________________

1890-91
POLITICAL ECONOMY 7.

Enrollment.

[Political Economy] 7. Professor [Charles Franklin] Dunbar. — Public Finance and Taxation. — Cohn’s Finanzwissenschaft. 3 hours.

Total 7: 2 Graduates, 4 Seniors, 1 Junior.

Source: Harvard University. Annual Report of the President of Harvard College 1890-1891, p. 58.

 

1890-91
POLITICAL ECONOMY 7.
Year-end examination (June 1891)

It is recommended that at least a third of the time be given to B.

A.

  1. Accepting the usual reasoning that a tax under some circumstances, by diminishing the income from property, diminishes its selling value, and so ceases to be felt by subsequent purchasers, should you say,—
    1. That the French impôt foncier is a tax on present landholders?
    2. That the English income-tax under Schedule A. is a tax on present landholders?
      The reason for the difference, if any exists.
  2. What is your final conclusion as to the sale of bonds or annuities at a discount, — is it defensible or not, and on what grounds?
  3. Explain the English method of using terminable annuities for the reduction of the public debt, as in 1867 and 1883, and discuss its advantages and drawbacks.
  4. “The administrator of local finances is permitted to found a sinking-fund at the time of issuing bonds, a permission, it will be remembered, contrary to sound rules of national financiering.”
    Does the distinction here made between local and national finances give solid ground for difference of treatment? Are the propositions as to the propriety of establishing sinking-funds in the two cases respectively tenable?
  5. Say remarks that the French government, in providing for the indemnity, bought any kind of foreign bills of exchange, “prenant tous les changes qu’elle pouvait acquérir sur quelque pay que ce fût.” How did this purchase of a bill, say upon Russia, facilitate the payment of the indemnity any more than the purchase of a bill upon Marseilles would have done, seeing that in either case the government had to collect the proceeds in order to use them?

B.

  1. In 1872 Herr Bamberger, in his pamphlet die Fünf Milliarden, regretted that Germany had not been allowed more time to absorb the indemnity, so as to avoid the risk of over-stimulated enterprise, rise of prices, and speculation.
    Discuss the probably effects, in Germany or elsewhere, (1) of longer time allowed to France; (2) of more cautious introduction of the wealth into Germany, effected

    1. By longer deposit (say) in London,
    2. By payment in securities, to be sold by Germany by degrees.

Source: Harvard University Archives. Harvard University, Examination papers, 1873-1915. Box 3. Papers Set for Final examinations in Philosophy, Political Economy, History, Roman Law, Fine Arts, and Music in Harvard College (June, 1891) in bound volume Examination Papers, 1890-92.

_____________________________

1890-91
POLITICAL ECONOMY 8.

Enrollment.

[Political Economy] 8. Mr. [William Morse] Cole. — History of Financial Legislation in the United States. 2 hours. 1st half-year.

Total 46: 1 Graduate, 29 Seniors, 13 Juniors, 3 Others.

Source: Harvard University. Annual Report of the President of Harvard College 1890-1891, p. 58.

 

1890-91
POLITICAL ECONOMY 8.
Mid-year examination

I.

  1. “That the public credit was much better during the Civil War than during the War of 1812 is proved by the fact that during the former war the bulk of the loans were sold at par, whereas during the latter the larger part were sold below par.”
    Comment on this statement.
  2. Under what authority does the Secretary of the Treasury purchase bonds in the market? Why does he not redeem them at par?
  3. Explain: “Five-twenties”; “Seven-thirties”; and tell why they were made seven-thirties; “Deferred sixes,” “5% redemption fund.”
  4. When were the first legal tenders under the present constitution authorized?
  5. What have been the express legal exemptions of government obligations from taxation since the outbrake of the Civil War?
  6. In what different capacities was each of the following men connected with the finances of the nation: Hugh McCulloch, Levi Woodbury, William Pitt Fessenden, John Tyler?

II.
(Omit two.)

1, 2. [Counts as two questions.] Compare the causes of the suspensions of 1814, 1837, and 1861.

3, 4. [Counts as two.] Mention the different systems which have been in vogue since 1789 for caring for public funds, and tell why each change was made.

    1. It was said that the bill which finally became the act for resumption in 1875 really provided for nothing in particular, and therefore ought not to be put upon the statute book. Did the provisions of the bill warrant the remark? Did subsequent history justify the remark?
    2. In what way were the French Spoliation Claims connected with the Second-Bank Struggle?
    3. Hamilton wished to have a system of internal taxation in working order as a resource in case of emergency. Does history throw any light upon the wisdom or folly of such a policy?
    4. What were the causes of Gallatin’s retirement from the Treasury?
    5. What history would you cite as an argument upon a proposition to replace bonds exempt from all taxation by bonds taxed to a moderate extent, and to distribute the receipts from the tax among the States according to the federal ratio?
    6. In a speech on the refunding bill it was said that the government made no threat, but merely promised certain privileges to those who presented bonds to be refunded at a lower rate. “No one proposes…the alternative adopted by our own Government under Hamilton’s plan of reducing the interest.”
      Is this implication regarding Hamilton’s funding justified by the facts?

11, 12. [Counts as two.] Mention as many as possible of the kinds of obligations which have been contracted by the government; i.e. character of obligation (bond or what), length of time to run, and rate of interest. Give one example under each kind,—giving period of issue, but not necessarily the exact date.

Source: Harvard University Archives. In Harvard University, Mid-year examinations, 1852-1943. Box 2. From bound volume Examination Papers, 1890-91.

_____________________________

1890-91
POLITICAL ECONOMY 9.

Enrollment.

[Political Economy] 9. Professor [Frank W.] Taussig. — Railway Transportation. —Lectures and written work. 3 hours. 2d half-year

Total 20: 14 Seniors, 4 Juniors, 2 Others.

Source: Harvard University. Annual Report of the President of Harvard College 1890-1891, p. 58.

 

1890-91.
POLITICAL ECONOMY 9.
Year-end examination (June 1891)

[Answer all the questions, and arrange your answers strictly in the order of the questions.]

  1. Can it be fairly said that the early experience of the States of the Union supplies strong arguments against State ownership of railways? Can it be fairly said of the experience of France since 1878?
  2. State the salient events in the history of the federal land grants to railways.
  3. How do you explain the rapidity with which the Union Pacific and Central Pacific railroads were completed?
  4. Wherein did the so-called Granger legislation on railway rates resemble the “natural” system advocated in Germany after 1871? Wherein did it differ from it?
  5. Sketch the history and machinery of the Southern Railway and Steamship Association.
  6. Explain the difference in the working of the Trunk Line Association before and after the passage of the Interstate Commerce Act.
  7. What ground is there for saying that the prohibition of pooling in the Interstate Commerce Act is inconsistent with its prohibition of discrimination between individuals?
  8. Suppose a railway to be built and used exclusively for coal traffic; would its rates be arranged on a plan essentially different from that in use with ordinary railways, having a varied traffic?
  9. It has been said that the principle of tolls, or rates based on cost of service, makes it necessary that each item of business should pay its share of the fixed charges. Why, or why not?
  10. Sketch the history of government management of railways in Italy.
  11. “Property has reached an ideal perfection. It is felt and treated as the national lifeblood. The rights of property nothing but felony and treason can override. The house is a castle which the King cannot enter. The Bank is a strong box to which the King has no key. Whatever surly sweetness possession can give is tasted in England to the dregs. Vested rights are awful things, and absolute possession gives the smallest freeholder identity of interest with the duke.”—Emerson, English Traits.
    Wherein does the trait here described make the railroad situation in England different from that in the United States?

Source: Harvard University Archives. Harvard University, Examination papers, 1873-1915. Box 3. Papers Set for Final examinations in Philosophy, Political Economy, History, Roman Law, Fine Arts, and Music in Harvard College (June, 1891) in bound volume Examination Papers, 1890-92.